SlideShare a Scribd company logo
1 of 202
Insuratest 2022
Welcome to the Insuratest P&C Exam Mastery System portion of your
INSURATESTexam preparation process;glad you are here. You will need
a few things before we begin this final section.Cinnamon gum, yes
cinnamon chewing gum, earplugs or headphones,and comfortable attire.
Seriously, go to the store and buy a pack of cinnamon chewing gum, a .50
cent pair of earplugs and find the headphones that came with your
smartphone; if you have noise canceling, one’s even better. (we will get to
this in a bit). If you are not yet dressed comfortablyand sitting in a relaxed
setting, please make that happen, then come back.
Please turn off,and tune out all outside and inside distractions; everything;
sports,reality tv, Facebook,texting, even that inner voice inside your head
that keeps on buzzing. You know, the one relentlessly stressing out about
the exam you’re about to ace! Let that voice take a nap.
Go to a quiet environment, or visit a comforting place like your favorite
coffee shop,conference room,or library. Go to where you know you will
focus the best. In reality, we don’t go to study; we instead move towards
learning. The deeperyou go towards studying, the more you will absorb
this section’s content and retain what’s necessaryto ensure you pass.
While in your preferred environment, try to block out all background noises
with headphones or earplugs. Yes, earplugs are the best.50 cents you will
ever spend on studying, far better and less expensive than a $4 latte’.
Using earplugs while learning lets you hear yourself think, and this is
paramount to your success.Earplugs prevent background noise from
interfering with your ability to move deepertowards studying and support
your ability to remain in study longer.
We want you to bring a pair of earplugs to your test site and wear them
during your exam. During the three-hour test, we do not want you to be
auditorily distracted by other test takers or proctors walking around, talking,
coughing, sneezing, or hearing sirens from the outside. Earplugs will
prevent outside noises from breaking your concentration.
Insuratest 2022
If you preferto listen to music, try listening exclusively to classical. Better
yet, music was written explicitly during and closelyfollowing the baroque
music era, years 1600 to 1750.Music written from this period carries a
distinct mathematically progressive time signature to its melodythat is
scientifically proven to boostbrain power when listened to while studying.
Listening to classicalmusic while studying will help you focus naturally and
will aid you in moving further towards study.
Sorry “today’s hits” you gotta go for now. If you have a commercial-free
Pandora radio account, try the “Classical for studying” station. Or put some
Bach, Vivaldi, Chopin, and Mozart on your iPod.
Get your cinnamon gum ready. Why Cinnamon? Chewing cinnamon gum
while studying has also been scientifically proven to boostbrain power,
narrow one’s focus,and stimulate the brain’s natural ability to tie a
fragrance to content memorization and enhance study material recollection.
Just smelling cinnamon boosts cognitive function and memory. We want
you to chew cinnamon gum while taking your State Exam quietly. You will
be amazed at how well it will help this course’s material circle back to you
on exam day.
Comfortable - Check
Earplugs or Headphones & Classical Music - Check
Cinnamon Gum - Check
Worries left to rest - Check
Insuratest 2022
Ok, let’s begin!
Hopefully, by now, you are relaxed, in your perfectstudy environment,
chewing cinnamon gum, listening to Mozart, and ready to dive right in. But
wait, we need to explain one more thing. The INSURATESTstudy theory.
Open your mind to this. It goes against all you have been pre-programmed
to acceptregarding educational principles and exam preparation
methodologies.We need to change the way you study and approach this
exam permanently.
Practice does notmake perfect; no, it does not. Practice makes
patterns.Practice the wrong pattern, and you will have to practice
approximately 85% more to achieve the same result naturally obtained by
simply practicing the correct pattern in the propersequence.
It is, however, the repetitive practice of executing a perfectpatternin
an ideal environmentthatmakes perfect.Belowis how you practice
perfectlyfor your State exam.
DO NOT TEST YOURSELF now.That’s right, DO NOT APPROACH THIS
AS A TEST because it’s a repetitive factual recognition and acceptance
exercise.Accepting this is paramount; the right answers are correct, they
are Bright and Bold, and right in front of you. All you need to do is read them
Backward and forwards, even out loud to yourself.Practicing this section
perfectlyas a tool will ensure you recall what is correct on exam day.
Insuratest 2022
Simply read the question(s) and the correctanswer(s). Then re-read the
correctanswer(s) and the question(s). If you want to get fancy, try and
make two questions out of each question in your mind by rewriting the
question using the answer and the original question as part of the answer.
Here is an example of the INSURATESTstudy theory in action.
(YOUR SCREEN IS NOT BROKEN. THE COLORS ARE THIS WAY BY
CHOICE, soon to be explained).
“Assignment” as an insurance term refers to
A. The agent assigned to an individual insured
B. The insurer’s right to subrogate a claim settlement from a third party
C. The transfer of an insured’s interest in a policy to another party
D. The insured’s right to change issuing company's mid-term without paying a short
rate cancellation fee
The same question rewritten in your mind
The transfer of an insured's interest in a policy to another party refers to
A. The agent assigned to an individual insured
B. The insurer’s right to subrogate a claim settlement from a third party
C. Assignment
D. The insured’s right to change issuing company's mid-term without paying a short
rate cancellation fee
A little secretabout“Practice Tests.”
Students are set up for failure by being forced to take standardized
insurance practice tests throughout educational development,at an
insurance school, in the pre-license training courses,or in a workbook. The
insurance exam simulator programs are no better. They all accomplishthe
exact wrong thing.
Insuratest 2022
By taking quizzes and tests with 10, 50, or even 150 questions on them,
students spend 85% of their time regardless,reading and studying wrong
answers, essentially inundated with erroneous statements and “noise.” It is
confusing you beyond confuzzlement while essentially trying to learn a new
language, the insurance language, in only fifty-two hours.
It’s not rocket science,just basic math. If you spend 85% of your time
looking at, reading to yourself,or out loud, essentially memorizing error-
filled statements,you’re more likely to recall what's wrong on exam day. Do
you have time for that?
No wonder it is the case that 61% of all who take this exam do not pass on
their first attempt. Could it be that they spent 85% of their preparation time
focusing on wrong answers? They could have spent time with family,
friends,working, or doing what makes them happy. Math doesn’t lie.
Did you know the answeryou picked,whetherrightor wrong,you are
90% more likely to recallon exam day? What if every answer you
rememberedwas the correctone? What if that was all you were re-
programmedto see?
Now we want you to accept what is right in the format presented here in the
Insuratest P&C Exam Mastery System. You must focus 100% of your time
absorbing and recognizing what is true and correct in the question format.
So that you will, with absolute certainty, recall what is right and accurate
when it matters mostwhen taking your state licensing exam.
Messagefrom our founder:
“I’ll let you in on a little secret.I did not pass this test the first time I took it.
Even after attending a ten-day, 52-hour insurance schoolclass, having a
great instructor, and studying for 80 hours plus, not passing was my result.
I was part of the 60% statistic.
Having to call my Dad, a successfulinsurance agent for thirty years at the
time, who had a desk waiting for me in his office since I had just graduated
college two weeks prior, to tell him I had failed my exam was one of the
hardest calls, I have ever had to make. (Waaaaay harder than cold calling).
Insuratest 2022
Even after spending most of my college graduation money to attend
insurance schooland “studying” for hours. I was among 61% left wondering
if this was even in the cards for me. It was a crushing disappointment.One
I desire for you to avoid.
When I finally figured out “how to study,” I immediately went to work on re-
engineering my study material to be viewed in the Insuratest P&C Exam
Mastery System format. I re-approached this exam with what is now the
INSURATESTstudy philosophy and my result on test day was staggeringly
different.
On exam day, I was the first to finish my exam in a room of 30 test takers. It
took me 92 minutes to finish a three-hour test. The same test I had run out
of time on two weeks prior. Taking my time, making sure every question
was answered correctly, I hit the submit button with absolute certainty that I
passed. (I ONLY SAW THE RIGHT ANSWERSON EXAM DAY,THEY
POPPED OUT FROM THE COMPUTERSCREENAS IF THEY WERE IN
BOLD FONT).
Walking out of the exam room on that breezy San Francisco afternoon with
a smile from ear to ear, doing a little fist pump, and honestly crying a few
tears of joy made for one of the bestafternoons of my life.
Helping you achieve this same success is our missionat INSURATEST.
Welcome again to the Insuratest P&C Exam Mastery System Sessions,
and thank you for choosing and trusting INSURATEST.com,“the insurance
test program designed to ensure you pass.”
Now let’s begin.
FOCUS ON WHAT IS RIGHT (Hint it’s Bold Black and Yellow ). What is
wrong is shaded lightly, so you won’t focus on or even read it. The material
only looks and feels like test questions,so you’ll respectthem as such. But
please,do not even read what’s not yellow or italicized bold;it’s only part of
the illusion.….
Now do yourselfa gigantic favor.Keep readingand prove the
INSURATEST study theory to yourself.
Insuratest 2022
Let’s start by quickly Mastering Single, Double,and Triple Negative
Questions.You can read/study as many questions as you want, but if a
double negative question’s phrasing habitually messes you up, you’ll pick
wrong answers on exam day. The question’s direction must be determined
first, even though you know the correctanswer. Scoring right hinges upon
your ability to interpret the question's true intent. This might be the
differencebetweenpassing and needing to try again.
Words and phrasesto look out for and whatis their
Mathematical/linguistic property,either negative (-) or positive (+).
Commit these to memory.
(-) Except, (-) Not, (-) Not Included, (-) Not True, (-) False), (-) None of the following,
(-) None of the above, are typically excluded (-).
(+) Included, (+) True, (+) Are part of the following, (+) Which of the following, (+)
Out of the following, (+) Of the following—single Negative Question
Examples.
Consequential loss is considered an indirect loss (as compared to losses
from the direct damage of a peril). Direct damages would be covered under
different types of insurance, such as property/casualty or fire insurance. All
of the following { are considered (+) } a consequential loss { except (-) }?
A. loss of rental value
B. damage to personal property
C. extra expense incurred while rebuilding damaged property
D. loss of business income while a building is closed for repairs
With single negatives think of them as a U-turn. Since the (-) negative word occurred at
the end of the question, the correct answer will be an untrue statement. (the exception,
one that is not like the others, the one that doesn’t belong). Remember the kid's TV
show Sesame Street’s “Which one is not like the others, which one just doesn’t belong”
song? I know it’s childish, but it works. Sesame Street can help you solve single
negative and double negative questions.
Insuratest 2022
{“Which one is not like the others, which one just doesn’t belong.” Translating a single
negative question is the same thing}
It was looking like this question’s answer would be in the form of a positive phrase (+)
until the last word except (-) came into play. Except caused the direction of the question
to take a U-turn. The last (-, or +) word is always the U-turn with single negative
questions.
Please be sure to read the entire question twice if need be. The word except was the
most important word to catch and was hidden in the end. With single negative questions
(-), the last word wins.
Extra expense insurance affords coverage for all of the following (+) if coverage is
in force at the time of loss except (-)?
A. Costs of setting up business in an alternative temporary location during the
period of restoration
B. Business income including payroll when a business is inoperable after
incurring a loss (Why? This is covered by business income, not Extra Expense
coverage).
C. The extra expenses a business incurs to continue a business while damage to
the premises is being restored
D. None of the above
Insuratest 2022
is Under a standard boat-owners policy. Which of the following liability
coverage(s) is/are typically excluded (-):
A. Vicarious liability
B. Flotilla liability
C. Medical liability
D. General liability
All of the following are part of (+) the standard business owners policy (BOP),
except (-):
A. Tenants improvements and betterments
B. Business personal property coverage
C. Professional liability coverage
D. Business income coverage
Double Negative Question Examples,
None of the following statements (-) differentiates the perils of robbery and
burglary in terms of coverage trigger except (-)?
A. Requires taking property directly from a person
B. Robbery is the broadest form of crime coverage
C. Burglary requires physical signs of forcible entry or exit
D. May involve use of force to cause bodily injury or instill a threat of violence
(Hint a negative (-) + a (-) = a Positive (+). This is true in both basic mathematics and
testing language. Since both negative words in this question (- None) and (- Except)
cancel each other out, you are looking for a positive phrase like “included” or, in this
case, a “true” phrase to answer this question. C. Burglary requires physical signs of
forcible entry or exit is the distinction and true phrase.
The coverage trigger between burglary and all other crime coverages is “forcible entry
or exit.” Burglary, by definition, “must be evidenced by physical entry or exit into a
Insuratest 2022
closed premise.” This is a rather easy definition question. Easy only if you navigated
through the double negative mine field correctly.
All of the following do not (-) describe a type of loss triggering a claim for
monetary damages due to an insured un-willfully causing injury to another
person or damage to another's property by a negligent act, except (-)?
Remember a (-) + a (-) = a positive, so you are looking for the term that matches the
definition stated in this question's text.
A. Real property loss
B. Physical loss
C. Personal property loss
D. Liability loss
Commercial property theft, also called “fidelity coverage,” is provided by none (-)
of the following coverage forms except (-)?
A. Commercial property form
B. Building and personal property coverage form
C. Commercial Crime Insurance
D. None of the above coverage forms afford coverage for theft
All of the following are not (-) covered by a Commercial General Liability policy
except (-)?
A. Damage to “Your Work”
B. Damage to “Your Product”
C. Contractual Liability
D. Premises operations
All of the following are inland marine policy exclusions (-), except (-):
A. Mechanical Breakdown
B. Earth movement
Insuratest 2022
C. Electrical breakdown
D. Wear and tear
Triple Negative Question Examples
The liquor liability exclusion contained in a Commercial General Liability (CGL)
policy excludes coverage (-) for none of the following (-) scenarios except (-)?
A. Serving alcohol at a company picnic
B. Buying a round of drinks for the sales staff at a company sponsored golf
tournament
C. The insured being involved in the alcoholic beverage business
D. None of the above
{Don’t worry about triple negative questions if they throw a few at you on exam day. Just
keep using basic math to help. Excludes coverage (-) for none of the following (-) These
cancel each other out, so now you are left with a single negative question except (-),
and those are easy.
CGL policies afford coverage for alcohol being served at company parties or corporate
events: as long as the insured is not directly involved in selling alcohol or being in the
alcohol service or distribution business. Therefore choice (C) represents the only
exclusion and is the correct answer}
All of the following are not (-) inland marine policy exclusions (-), except (-):
A. Mechanical Breakdown
B. Earth movement
C. Electrical breakdown
D. Wear and tear
All of the following are not (-) inland marine policy exclusions (-), except (-):
are not (-) + policy exclusions (-) = a +, which is then negated by the
third negative ( except (-) making this in the end just another basic “except” (-) question.
Insuratest 2022
Do not guess on single, double,or even triple negative questionsor
succumbto their trickery.
Use basic math to help you solve single or triple negatively phrased questions designed
to confuse you. The key is to know which direction you are being asked to answer in the
end, either - or +. Use your scratch paper to keep track of phrases to add, subtract, or
cross out. Figure out your end direction, then simply choose the correct answer; it’s that
easy.
Good news
Honestly, most double negatively phrased questions are easier content-wise than
straightforward questions. The test writers only turn the phrasing into that of a single,
double or triple negative in an effort to toughen up what would otherwise be an easy-to-
average question.
All of the following are covered (+) under the unendorsed ISO building and
personal property coverage form except (-)?
A. Light pole
B. Gate
C. Patio
D. Signs
All of the following are not (-) an additional coverage provided by an unendorsed
HO-3 policy except (-)?
A. Tenant legal liability
B. Lawn, trees, shrubs and other plants
C. TRIA
D. Safe Burglary
An agent’s appointment to an insurance company is terminated and the agent
has no other appointments with any other insurance company at this time. Which
of the following happens to the agent’s license automatically?
A. The agent’s resident license remains in force until its term expires.
Insuratest 2022
B. The agent’s resident license will be terminated and they will have to sign it over
to the insurance commissioner
C. The agent’s resident license becomes inactive.
D. The agent's license will simply expire upon renewal.
All of the following are (-)not required of a producing broker-agent before
appointing an insurance solicitor (-)except?
A. Must be an MGA
B. Must hold a permanent license
C. Must submit a notice of hiring to the DOI in 4 business days or less
D. Must possess a certificate of authority
It is proven that another person's negligent actions contributed to an injury What
gives an insured the right to seek compensation?
A. Coinsurance
B. Tort law
C. Waiver of subrogation
D. Reinsurance
Consequential loss is considered an indirect loss (as compared to losses
from the direct damage). Direct damages would be covered under different
types of insurance, such as property/casualty or fire insurance. All of the
following are considered (+) a consequential loss except (-)?
A. Loss of earnings
B. Damage to personal property
C. Extra expenses incurred while demolition of a partially burned structure
Insuratest 2022
D. Extra expense incurred to move a business to a temporary location
All of the following are true (+) of an Insurance Adjuster?
A. Assists in settling claims on behalf of an insurance company
B. Typically an employee of an insurance company therefore does not need a
license
C. Must be licensed
D. Both A and B
All of the following are true (+) of a Public Insurance Adjuster?
A. Assists an insured in settling claims with an insurance company
B. Typically an employee of an insurance company therefore does not need a
license
C. Must be licensed
D. Both A and C
Which of the following are (+) exclusive licenses?
A. Bail
B. Title
C. Mortgage Guarantee
D. All of the above
Insuratest 2022
Special lines Surplus lines brokers:
A. Transact business only with (represent )non-admitted insurers and must
post a $50,000 Bond.
B. Hold a valid surplus lines insurance license
C. Transact business exclusively with foreign insurers
D. Specialize in large commercial accounts only
All of the following insurer settlement options under a personal auto policy are
not available (-) except (-)?
A. Induce the insured to purchase a replacement auto from a wholesaler
B. Recommend the insured first attempt to settle with the negligent operator’s
insurance as to avoid subrogation.
C. Return stolen property to the first named insured at the address shown on
the policy declarations page
D. Demand the insured repair their vehicle at a repair shop on the insurer’s
preferred provider list.
All of the following are not (-) a common policy conditions for a commercial
package policy except (-)?
A. Waiver of subrogation
B. Why are you reading this
C. Pro-rata clause
D. Inspections and surveys
Which of the following is true (+) of an insurance agent?
A. Represents the company and sells insurance
Insuratest 2022
B. Must be appointed and may have binding authority.
C. Cannot transact life or disability insurance.
D. All of the above
Shipowners may acquire coverage against vicarious liability claims due to the
negligent operation of their vessel by their employee(s); which of the following
coverage forms?
A. Yacht owners policy
B. Protection and indemnity insurance
C. Ocean Marine coverage
D. Longshoreman Indemnity act
Which advisory organization(s) develop forms for the standard market?
A. Acord Form processing service (AFPS)
B. National Association of Insurers and Financial Advisors (NAIFA)
C. Insurance Services Office (ISO)
D. National Association of Insurance Commissioners (NAIC)
Under the common policy conditions, who is authorized to make changes to a
commercial policy?
A. The producer
B. The first named insured (with insurer's consent)
C. Any insured named on the policy
D. The insurance company
None of the following are true (-) regarding the Law of Large Numbers except (-)?
A. The larger the number of similar risks that are combined in a group, the
more predictable future expected losses would be
Insuratest 2022
B. Applicable use of the Law of Large Numbers is a characteristic of an ideally
insurable risk
C. Cannot be used to predict individual losses or profits
D. All of the above
The Insurance Commissioner-elect uses which of the following as a means to
determine whether or not an insurer’s filed rate is excessive or unjustly
discriminatory?
A. Whether the proposed filed rate mathematically reflects the insurance
company’s investment income
B. The length of time passed since an insurer’s last full examination of its books and
records by the office of the insurance commissioner
C. If their rates are comparable to other insurers with the same business model
D. In accordance with the guidelines set forth by the FDIC
In comparing the personal property coverage of a dwelling policy to that of a
standard ISO homeowners policy form, deduce which of the following is not (-)
false (-)?
A. Why are you reading this?
B. Homeowners' policy claims are settled on an actual cash value basis, while
dwelling policy claims are settled on a replacement cost basis
C. There is no appreciable coverage difference
D. Homeowners' off-premises coverage is 10% of the total property limit, while that
of a dwelling policy is 100%.
All of the following perils are not covered (-) on the basic form except (-) ?
A. Terrorism
B. Riot or civil commotion
Insuratest 2022
C. Collapse
D. Vandalism
Which of the following statements best defines the standard commercial property
policy “concurrent causation exclusion” and the effect it has on a potential claim
settlement:
A. Why are you reading this,
B? If a loss occurs during the 90-day Binder issuance period, it is excluded
C. If two or more perils happen in sequence to cause a loss, then only the damage
caused by the first peril is covered
D. If two or more perils happen at the same time to cause a loss, there is
coverage for all of the damage if any of the perils are not specifically excluded
Brian a landscape architect is brought into a suit claiming his design is solely
responsible for the damage caused by improper draining at a claimant's property.
What type of insurance would protect Brain For this type of mistake even if it is
determined later that the suit is groundless?
A. Professional liability
B. Errors and Omissions insurance
C. Why are you reading this
D. Fidelity Bond
How does an insured's unendorsed personal auto policy respond in the event an
auto accident occurs in Florida a no-fault state that requires personal injury
protection (PIP)?
A. PIP coverage is provided, but the limit moves to Florida’s state minimum
B. Coverage is excluded without the intestate travel endorsement
C. Coverage automatically is broadened to provide PIP
Insuratest 2022
D. The insured must purchase PIP from a licensed Florida agent within three
business days of the accident if at fault ten days if not at fault.
The named non-owner coverage endorsement form adds which of the following
coverage(s)?
A. Liability, medical payments, and uninsured motorist
B. Other than collision, vicarious liability, and medical payment
C. Bodily injury, medical indemnity, and other than collision
D. Property damage only
Ariana leaves her vehicle at an auto dealer's service center overnight to complete
her scheduled maintenance. The following day a mechanic employed by the
dealership makes a “Grande” mistake and is involved in an auto accident while
test driving Ariana’s car. What coverage if any will Ariana’s personal auto provide
for this claim?
A. The personal auto policy is primary
B. The personal auto policy is excess
C. Coverage is excluded under the personal auto policy
D. Why are you reading this
All of the following are not true (-) except (-) when comparing the per accident
coverage limit difference(s) between auto policies with single limits vs. split
limits?
A. The per accident single limit covers to all claims, while the split limit covers
medical payments exclusively
B. The per accident single limit applies only to property damage, while the split limit
applies to vicarious liability exclusively
Insuratest 2022
C. The per accident single limit applies to property damage, while the split limit
applies to bodily injury exclusively
D. There is no difference
All of the following are not considered (-) special damages except (-)?
A. Compensation for time away from work
B. Compensation for a claimant's pain and suffering
C. Compensation for a claimant's loss of hearing
D. Compensation for a claimant's loss of limb
All of the following are part of (+) the standard business-owners policy (BOP),
except (-):
A. Tenants improvements and betterments
B. Business personal property coverage
C. Professional liability coverage
D. Business income coverage
Use this chart to help illustrate the different insurer Cancellation, Renewal and
Nonrenewal notices as well as the advance “days” notice they require for
Personal Auto, Homeowners and Commercial insurance questions. Test your
recollection with the following questions.
Personal Auto Homeowners Commercial
Cancellation for
Non-payment
10 days 10 days 10 days
Notice of Non-
Renewal
30 days 30 days 60 days
Notice of Renewal 20 days 30 days 30 days
Insuratest 2022
An insurer decides to non-renew a commercial auto policy how many days written notice
must they provide the insured?
A. 10
B. 20
C. 30
D. 60
An insurer decides to renew a commercial auto policy how many days written notice
must they provide the insured?
A. 10
B. 20
C. 30
D. 60
An insurer decides to non-renew a personal auto policy how many days written notice
must they provide the insured?
A. 10
B. 20
C. 30
D. 60
An insurer decides to renew a personal auto policy how many days written notice must
they provide the insured?
A. 10
Insuratest 2022
B. 20
C. 30
D. 60
An insurer decides to non-renew a Homeowners policy how many days written notice
must they provide the insured?
A. 10
B. 20
C. 30
D. 60
An insurer decides to renew a Homeowner policy how many days written notice must
they provide the insured?
A. 10
B. 20
C. 30
D. 60
An insurer must afford a personal auto policy owner _______ days written notice of
cancellation for non-payment of premium?
A. 10
B. 20
C. 30
D. 60
An insurer must afford a Homeowners policy owner _______ days written notice of
cancellation for non-payment of premium?
A. 10
B. 20
C. 30
Insuratest 2022
D. 60
An insurer must afford a commercial policy owner _______ days written notice of
cancellation for non-payment of premium?
A. 10
B. 20
C. 30
D. 60
Of the various types of cancellations which one occurs when the insurer keeps
the earned, and returns the unearned premium?
A. Pro-rate/rata
B. Flat Rate
C. Rescind/Recission
D. Short Rate
Of the various types of cancellations which one occurs when the insurer keeps
the earned,and returns the unearned minus a fee for admin charge or overhead
expense?
A. Pro-rate/rata
B. Flat Rate
C. Rescind/Recission
D. Short Rate
Of the various types of cancellations which one occurs when the insurer
voids the contract and returns all premiums?
A. Pro-rate/rata
B. Flat Rate
Insuratest 2022
C. Rescind/Recission
D. Both B and C
{Flat Rate and Rescind/Recission mean the same thing: when the insurer
Voids the contract and returns all premiums as if the contract never existed}
An agent who redirects premium funds held in a fiduciary account to his or her
personal account is guilty of?
A. Fraud
B. Theft
C. Falsifying
D. Perjury
{Regarding Fiduciary responsibility please note that the term Fiduciary refers to
premiums held in trust. Any form of commingling or embezzling is a form of theft.
Please also remember that premium refunds coming from the company must be
returned within 25 days. While premium refunds coming from the agent of record must
be returned within 15 days.
None of the following are true (-) of an insurance broker except (-)?
A. An Insurance Broker represents the client and secures coverage
B. Must post a $10,000 bond
C. Cannot bind coverage or transact life or disability insurance
D. All of the above
Standard commercial lines insurance policies contain Interline endorsements
(such as cancellation notices or nonrenewal provisions). These apply or could
apply to:
A. More than one coverage part of a package policy
B. Why are you reading this
C. To liability coverage only
Insuratest 2022
D. Only to waivers of subrogation issued to certificate holders by endorsement
All admitted insurers are required to maintain an investigative unit to question
which of the following?
A. Possible non-actuarially based discrimination
B. Unfair employment and hiring practices
C. Possible fraudulent claims submitted by insureds
D. All vicarious liability claims
Under a standard boat-owners policy. Which of the following liability coverage(s)
is/are typically excluded (-)?
A. Vicarious liability
B. Flotilla liability
C. Medical liability
D. General liability
The vacancy permit endorsement covers which of the following perils?
A. Vandalism
B. Building glass breakage
C. Sprinkler leakage
D. All of the above
None of the following (-) are considered special damages except (-)?
A. Loss of hearing
B. Mental anguish
Insuratest 2022
C. Lost wages
D. Reassignment
Insureds Lori and Daniel rent a convertible while vacationing in hawaii what
coverage, if any, will respond to a hypothetical physical damage claim under their
personal auto policy?
A. They need to purchase physical damage coverage at the car rental agency
B. Why are you reading this
C. No coverage available
D. The broadest coverage on any auto shown in the declarations of Daniel &
Lori’s personal auto policy
All of the following are not (-) one of the principal rating factors under Proposition
103 permitted for personal auto policies except (-)?
A. Merit rating
B. Safety driving record
C. Credit report
D. Using prior policy as a method of rate factoring
Which of the following is the defining characteristic of a Fraternal Insurer?
A. Charitable
B. Non-participating policies
C. Owned by shareholders
D. Shareholders receive taxable dividend (when declared)
None of the following (-) are eligible for a Businessowners policy except (-)?
A. An auto dealer
B. A furniture store
C. A casino
Insuratest 2022
D. A banking institution
Which of the following is not listed (-) on a Commercial General Liability
declarations page?
A. Pollution liability limit
B. General Aggregate Limit
C. Rented to you sub-limit
D. Personal and advertising injury limit
None of the following (-) are forms of Professional liability insurance except (-)?
A. Medical Malpractice
B. Errors and Omissions
C. Directors & officers and Employer Practices Liability
D. All of the above
Subrogation as an insurance term is summarized by which of the following
statements?
A. The insurance policy liability limits respond separately to each claimant
B. The insurer’s right to recover payment from a responsible third party
C. The insured’s right to receive PIP coverage at no extra charge while traveling
across state lines
D. The transfer of insurable interest in a policy to another party when a vehicle is
sold
{Remember subrogation is basically substitution. It is the transference of an insured's
claim rights of financial recovery, to a third party. Ex: an insured is rear ended. Their
insurance company (insurer) settles the claim, even though the accident was proven not
to be their insureds fault. After paying said claim, the non-fault driver’s insurer
(insurance company) then asks their insured to sign a notice of subrogation that
basically says: “I have been paid (indemnified) for this claim (my loss) and I sign over
my rights of subrogation (right to collect again) to my insurance company.”
Insuratest 2022
This is done so that the non-fault representing insurer, who has paid out on said claim,
may then be paid back through subrogation, by the at-fault driver's insurer.
By the insured (non-fault driver) signing over their claim rights after being indemnified by
their insurer, the principle of indemnity is followed. For by signing over rights of
subrogation after indemnification (being paid for a loss) prevents an insured from
collecting twice on a claim. So in essence subrogation supports indemnity, the principle
of insurance}
An insured submits a claim for stolen jewelry and signs the claim form. Later the
claim is proven to be fraudulent. The insured may be guilty of_____?
A. Attempted theft
B. Collusion
C. Perjury
D. An insurance related misdemeanor
All of the following statements regarding business owners policies is/are true (+)
except (-)?
A. Insureds have the benefit of additional coverages that might otherwise be
overlooked
B. Combine coverage for special advantages
C. Assembles the basic coverages required by a business owner in one bundle.
D. All of the above
_________________ are very specific as far as money in a claim settlement is
concerned. Example: It is exactly known how much a car door costs to replace,
including labor.
A. Stated Values
B. Specific Awards
C. General Damages
D. Special Damages
Insuratest 2022
Which of the following deals with lost wages or salary?
A. Punitive Damages
B. Specific Award
C. General Damages
D. Special Damage
Non-specific losses, with non-specific claim settlements such as pain and
suffering or mental anguish are characteristic of?
A. Punitive Damages
B. Specific Award
C. General Damages
D. Special Damage
Awards using a monetary punishment to set an example to the industry?
A. Punitive Damages
B. Specific Award
C. General Damages
D. Special Damage
{Example: In 1992, 79-year-old Stella Liebeck bought a cup of coffee at a McDonald’s
drive-thru in Albuquerque New Mexico and spilled it on her lap after opening it up to put
cream and sugar in it. She sued McDonald’s and a jury awarded her nearly $3 million in
punitive damages for the burns she suffered. Stella definitely got her groove back with
that settlement! Ever notice now at the McDonalds drive through they ask if you want
cream and sugar at time of order, so they may put it in for you. Risk management
effectively eliminated the need for their customers to ever open their hot coffee cup lids,
especially while driving.}
Insuratest 2022
Which of the following items of personal property are covered by an unendorsed
personal auto policy and if so up to what limit?
A. Electronic items up to $500
B. Non commercial use electronic equipment up to $1500
C. All personal property left in vehicle owned by the first named insured up to $500
D. There is no coverage for personal property
The following are characteristics of a Stock Insurer (+)?
A. Non-participating policies
B. Owned by shareholders
C. Shareholders receive taxable dividend (when declared)
D. All of the above
Which of the following is the defining characteristic of a Reciprocal Insurer/Inter-
insurance Exchange?
A. Also known as participation policies
B. Owned by policyholders
C. Policyholders receive tax-free dividends (when declared)
D. Unincorporated/Attorney-in-fact
Lloyds of London is a marketplace with physical facilities where business is
conducted by groups of Lloyd’s members known as ___________?
A. Captives
B. Wholesales
C. Syndicates
D. intermediaries
Insuratest 2022
None of the following are true (-) of Mutual Insurers except (-)?
A. Also known as participation policies
B. Owned by policyholders
C. Policyholders receive tax-free dividends (when declared)
D. All of the above
All of the following are inland marine policy exclusions (-), except (-):
A. Mechanical Breakdown
B. Earth movement
C. Electrical breakdown
D. Wear and tear
A home’s foundation is damaged during an earthquake, minutes later a gas line
ruptures causing an explosion. The home burns to the ground. Which coverage if
any is afforded by the unendorsed HO-3 policy form in force at the time of loss?
A. There is no coverage for damage to the foundation directly caused by the
earthquake, only for the damage caused by the explosion and subsequent fire
B. There is no coverage for any loss directly caused by an earthquake in the
unendorsed HO-3 policy form
C. The HO-3 is a named peril policy form and does not cover earthquake or
damages caused by explosion.
D. The policy covers all of the damage since explosion is an insured peril
At what time during the sales process is a fire and casualty agent required to
inform a prospective insured of any conditions which may be precedent to
coverage regarding a homeowners policy?
A. At quote
B. At binding
Insuratest 2022
C. At application
D. At policy issuance
The insurer’s right to recover its claim settlement payment to their insured from a
negligent third party is the definition of the following term:
A. Subrogation
B. Inherent claim rights
C. Arbitration
D. Reassignment
What is the minimum discount awarded to "good drivers" as a percentage for
meeting the qualifications?
A. 10%
B. 15%
C. 20%
D. 25%
Interline endorsements such as notices of cancellation may pertain or could
pertain:
A. To more than one coverage part of a package policy
B. Exclusively to ocean marine coverage
C. To general liability and advertising injury only
D. Only when required by a certificate holder
An armored vehicle transporting an insured's bank deposit is robbed at gunpoint.
In order for the insured's loss to be covered what type of insurance would need to
be in force at time of loss?
A. Safe burglary
B. Inland marine coverage form
C. Outside the premises-theft of money and securities
Insuratest 2022
D. Robbery
10 days prior to cancellation for non-payment of premium a written notice is
mailed to the insured by the insurer. Which of the following requirements
regarding written notices by mail from an insurer to an insured is not required (-)?
A. Notice must be sent with return receipt required
B. Notice must be mailed to the residence or principal place of business
C. Postage must be prepaid
D. Why are you reading this
In anticipation of an upcoming vacancy period at a seasonal business location
the vacancy permit endorsement is added to a building and personal property
coverage form. The basic policy's vacancy conditions as an interline
endorsement is/are:
A. Waived during the policy period
B. Waived only for designated perils during the permit period
C. Waived only for general liability claims during the policy period
D. Waived during the permit period but only if a security system is installed
What is the primary coverage differential between umbrella and excess liability
policies?
A. The umbrella has must follow form and be issued by the same character as the
general liability policy
B. The umbrella covers physical damage losses only not general liability losses
Insuratest 2022
C. An umbrella policy may include coverage for perils not covered by the
underlying policy; subject to a self insured retention limit (SIR). While excess
liability policies follow form and may offer monoline coverage.
D. The umbrella affords defense costs only if the insured wins the suit the excess
liability does covers defense costs regardless
In which of the following situations is off-premises coverage void under the
business personal property coverage form?
A. In a salesperson’s luggage
B. While in a delivery truck
C. At a convention center
D. In a salesperson’s vehicle
Which of the following policy forms affords an insured coverage from any loss
except that of which arises from specifically excluded perils?
A. Specified cause of loss
B. Stated Value
C. All-risk, Special Form, or open peril
D. Named Peril
The basic boat owner's policy form is similar to what other policy in terms of
coverage?
A. The personal auto policy
B. A yacht owners policy
C. A dwelling policy
D. An inland marine floater policy
Insuratest 2022
Please get up, walk around,come back in 10-20 min to keep it fresh.
Vito’s Pizzeria owner and first named insured Vito Corleone does not own the
building his pizzeria operates out of. Any improvements and betterments he
makes to the building he leases are covered under what section of his (BOP)
business owner's policy?
A. Inland Marine
B. Business personal property
C. Rented to you sublimit
D. “Forget about it”
Insuratest 2022
A loss ratio is calculated by which of the following formulas?
A. Expenses divided by premiums
B. Losses and expenses divided by premiums
C. Losses divided by premiums
D. None of the above
An expense ratio is calculated by which of the following formulas?
A. Expenses divided by premiums
B. Losses and expenses divided by premiums
C. Losses divided by premiums
D. None of the above
A combined ratio is calculated by which of the following formulas?
A. Expenses divided by premiums
B. Losses and expenses divided by premiums
C. Losses divided by premiums
D. None of the above
Which of the following are reasons Insurer’s purchase reinsurance:
A. Unearned premium reserve reduction
B. To protect themselves from catastrophic losses, assisting in profit
stabilization
C. Increases the insurer's underwriting capacity
D. All of the above
Which if the following is true with regard to private passenger vehicle inspection
requirements outlined in the insurance code?
Insuratest 2022
A. Requires a vehicle inspection only if both comprehensive and collision coverages
are carried
B. The insurance code requires a vehicle inspection for any vehicle with a gross
weight exceeding 4,000 pounds
C. The insurance code makes no mention of a vehicle inspection requirement
D. No vehicle inspection is required if coverage D is added at policy inception
What coverage if any would respond to an accident involving a named insured
driving a company car who carries an unendorsed personal auto policy?
A. Coverage (B) Medical payments
B. The unendorsed policy form excludes this usage
C. Why are you reading this
D. Coverage (A) General Liability
Claims for legal liability may stand on the basis of all of the following (+) except (-
)?
A. Breach of contract
B. Vicarious liability
C. Tort law
D. Negligence
With regard to admitted insurers actuarial methodologies, all of the following are
not true (-) except (-)?
A. They may use race, color, religion, or national origin as a condition to apply
higher rates as long as their means are actuarially sound
B. They may not use race, color, religion, or national origin as a condition to
apply higher rates
C. They may use any actuarially sound method when rating policies as long as they
can be statistically proven
D. They may use any actuarially sound method when rating policies as long as they
can be statistically proven and are filed with the department of insurance
Insuratest 2022
Which of the following statements best defines policy “assignment”:
A. A transfer of legal rights under, or interest in, an insurance policy to
another party.
B. A transfer of legal rights under, or interest in, an insurance policy to only the first
named insured
C. The insurer’s right to collect damages from a third party only if a waiver of
subrogation endorsement is in force
D. The insured’s right to change insurance companies mid term without having to
pay a new policy fee
What effect does the building ordinance or law exclusion/limitation have on both
commercial and homeowner’s property coverage policies?
A. It automatically voids replacement cost coverage for both commercial and
residential structures built built before the year 1985
B. It excludes that part of the loss resulting from the enforcement of a
building ordinance or law for commercial buildings and limits it’s allocated loss
dollar by a percentage of total property coverage limit on homeowner’s policies.
C. It excludes that part of the loss resulting from the enforcement of a building
ordinance or law or and limits it’s allocated loss dollar by a 5% of total liability coverage
limit.
D. Does not include coverage to upgrade handicap access building code
enforcement
While property insurance covers debris removal for a portion of property
damaged by a covered peril, it doesn’t cover demolition expenses for an
Insuratest 2022
undamaged portion of a building that has to be removed. Which Ordinance or
Law coverage part provides this coverage?
A. Part A of Ordinance or Law provides this coverage.
B. Part B of Ordinance or Law provides this coverage.
C. Part C of Ordinance or Law provides this coverage.
D. Part D of Ordinance or Law provides this coverage.
Which of the following statements differentiates the perils of robbery and
burglary in terms of coverage trigger?
A. Requires taking property directly from a person
B. Robbery is the broadest form of crime coverage
C. Burglary requires physical signs of forcible entry or exit
D. May involve use of force to cause bodily injury or instill a threat of violence
Coverage for which of the following is provided by coverage(s) A and/or B of the
ISO commercial general liability (CGL) policy form?
A. Vicarious liability
B. Workers compensation statutory limit
C. Pollution Liability
D. Advertising injury
Which type of loss describes that of a business exposed to the death, injury or
disability of employees?
A. Personal loss exposure
B. Personnel loss exposure
C. Fraternal loss
D. nolo contendere
Insuratest 2022
An Ohio based companies only option available for the purchasing workers
compensation insurance is through the Ohio Bureau of Workers Compensation.
Which of the following statements is true of Ohio’s workers compensation:
A. Ohio has a competitive state fund
B. Ohio has a shared market fund
C. Ohio has a monopolistic state fund
D. Ohio has a guaranty fund
The act of an insurer making a false statement either written or oral with the
intent to deceive to any examiner lawfully appointed to examine the insurer’s
books and records is defined by which of the following?
A. A barring act
B. Concealment
C. An unfair practice
D. Fraud
Which of the following is true of extra expenses under the business income
coverage form?
A. Excluded
B. Covered without limit during the period of restoration
C. Payments are executed if the restoration period exceeds 6 months
D. Covered up to 25% of the policy limit specified in the declarations
As an insurance agent which endorsement would you recommend your
insured add to their personal auto policy to obtain liability coverage for their all-
terrain vehicle (ATV)?
A. Coverage is only available through the purchase of a recreational vehicle stand
alone policy
B. ATV Flotilla liability
Insuratest 2022
C. Miscellaneous type vehicle endorsement
D. Recreational/seasonal vehicle endorsement
What legal recourse in action, in terms of claim settlement allotment does an
injured party to a contract have if the other party is found to be in breach of said
contract?
A. Claim rights only if a waiver of subrogation is in force prior to execution of said
contract
B. Damages for breach, reasonable attorney fees and costs
C. Damages for breach, reasonable attorney fees and costs less the self insured
retention limit
D. Damages for breach exclusively
Inherently possessing the possibility of financial loss resulting from ownership of
property is known as?
A. The principal of insurance
B. Forecasting
C. Insurable interest
D. Adverse selection
(All of the following (+) are representative of areas market regulation covers,
except (-)?
Following the rule you are seeking a negatively phrased outcome (The “exception” in
this case the one which does not belong :
A. Employee training
B. Monitoring and preserving the financial solvency of insurance companies
C. Regulating and standardizing insurance policies and products
D. Controlling market conduct and preventing unfair trade practices
Insuratest 2022
Protection and Indemnity insurance, covers all of the following (+) occurrences
except (-)?
A. Loss of life, injury and illness of crew, passengers and other persons
B. Damage to docks, buoys and other fixed and floating objects
C. Mutiny and misconduct by crew
D. The cost of reasonable measures an insured must take to prevent or
mitigate damage at time of loss
Which of the following occurrences would be covered if an insured purchases a
scheduled personal property endorsement (i.e.: personal articles floater) from his
agent to append to their already in force previously un-endorsed homeowners
policy form?
A. Wear and tear, gradual deterioration, latent defect, insects or vermin
B. Inherent vice
C. Breakage of glassware caused by an earthquake
D. Personal property on display at a county fair
Coverage for transportation expenses in an unendorsed personal auto policy
form is provided in the event __________________ losses occur and coverage is
in force at time of loss?
A. Theft losses only
B. Other than collision losses only
C. Collision and other-than-collision losses
D. Transportation expenses are not available in the unendorsed policy form
An implied warranty in an insurance contract qualifies as which of the following?
Insuratest 2022
A. An representation
B. An Oath
C. Res Ispa Loquitur (It speaks for itself)
D. A Jurat
{Remember a representation is the best of one’s knowledge or belief/promise.}
None of the following (-) are characteristics of a Representation except (-)?
A. Representations may be either oral or written.
B. A Representation may qualify as an implied warranty.
C. A representation may be changed before the policy goes into effect and
agrees with its assertions or stipulations
D. All of the above
Which of the following is/are true of a/an Estoppel?
A. An Estoppel essentially removes a real or potential liability for the other party in
the agreement.
B. In a settlement between two parties, one party might, by means of an Estoppel,
relinquish its right to pursue any further legal action once the settlement is finalized.
C. The Estoppel can either be in written form or some form of action. An Estoppel
essentially removes a real or potential liability for the other party in the agreement.
D. A legal doctrine restraining a party from contradicting its own previous
actions if those actions have been reasonably relied on by another party.
{Example: An insurer that has habitually accepted late premium payments from an
insured may be estopped from later canceling said policy on the grounds of
nonpayment because the insured has been reasonably led to believe that late payments
are acceptable.}
Which one of the following is characteristic of the the loss prevention method
“Risk Avoidance”?
A. A homeowner decides to buy a trampoline
B. A homeowner installs carbon monoxide detectors
C. A new homeowner decides to fill in the backyard swimming pool
Insuratest 2022
D. A homeowner purchases flood insurance
What is the coverage limit specific for tapes, records, disks, and other media in
the “Coverage for excess sound producing equipment, audio, visual and data
electronic equipment, and tapes, records, discs and other media endorsement” is
found in a personal auto policy (PAP)?
A. $500
B. $200
C. Coverage is
D. $1,000
Insuratest 2022
Coverage in the inland marine policy form contains all of the following perils as
exclusions (-) , except (-)?
A. Damage from insects, vermin or rust
B. Wear and tear
C. Earth movement
D. Inherent vice
Which of the following is/are true of a waiver?
A. A waiver essentially removes a real or potential liability for the other party
in the agreement.
B. In a settlement between two parties, one party might, by means of a waiver,
relinquish its right to pursue any further legal action once the settlement is
finalized.
C. The waiver can either be in written form or some form of action. A waiver
essentially removes a real or potential liability for the other party in the
agreement.
D. All of the above
{ At the most basic level a Waiver effectively gives up a right}
All of the following (+) will influence an entity's ability to hold a license except (-)?
A. Resignation by or termination of a key employee
B. Cessation of a co-partnership
C. Cessation of an LLC
D. Cessation of an association
Which of the following best defines a claim loss reserve as it relates to the
insurer?
Insuratest 2022
A. The total incurred amount less the total paid amount
B. The total paid amount less the total paid amount
C. An estimate of the value of a claim or group of claims not yet paid
D. Losses in excess of policy limits less self insured retention fee
Which of the following defines a case reserve as it relates to the insurer?
A. An estimate of the amount for which a particular claim will ultimately be
settled or adjusted or reconciled
B. The total incurred amount less the total paid amount
C. The total paid amount less the total paid amount
D. Losses in excess of policy limits less self insured retention fee
The miscellaneous type vehicle endorsement when added to a personal auto
policy affords an insured which of the following?
A. Liability and Medical Payments coverage exclusively
B. Physical damage coverage exclusively
C. Passenger Liability exclusively
D. All coverage under the personal auto policy
None (-) of the following define Market Conduct Regulation except (-)?
A. Examiners from the Market Conduct Regulation sector physically audit insurance
carriers only to review their business practices as they relate to licensing, claims, tax
payments, rates and form approval, advertising and sales aids, underwriting and rating,
policyholder service, and complaint handling
B. Examiners from the Market Conduct Regulation sector physically audit
insurance agencies, insurance companies, etc., to review their business practices
as they relate to licensing, claims, tax payments, rates and form approval,
Insuratest 2022
advertising and sales aids, underwriting and rating, policyholder service, and
complaint handling.
C. Examiners from the Market Conduct Regulation sector physically audit non-
admitted insurance companies exclusively, to review their business practices as they
relate to licensing, claims, tax payments, rates and form approval, advertising and sales
aids, underwriting and rating, policyholder service, and complaint handling
D. None of the above
Which of the following best defines “liberalization clause” and is accurate in
terms of it’s sectional placement in an insurance policy?
A. In umbrella liability insurance, a clause specifying that coverage will be as
broad as that provided by the primary liability policies, without additional
premium, found in the policies declarations section. (I.E. policy follows form)
B. In umbrella liability insurance, a clause specifying that coverage will be as
broad as that provided by the primary liability policies, without additional
premium, found in the policies insuring agreement section. (I.E. policy follows
form)
C. In umbrella liability insurance, a clause specifying that coverage will be as
broad as that provided by the primary liability policies, without additional
premium, found in the policies conditions section. (I.E. policy follows form)
D. None of the above
How far in advance must an insurer provide written notice to an insured before it
may legally cancel for non-payment of premium?
A. 4 days
B. 10 days
C. 25 days
D. 30 days
Which of the following is true of an agent who possesses a valid inactive
California insurance license?
A. The agent may review quotes with a prospect but does not have binding authority
Insuratest 2022
B. The agent may not transact insurance as there is no notice of appointment
on file with the department of insurance but may keep their license inactive if
their licensing fees are paid and all continuing education requirements are met.
C. Why are you reading this
D. None of the above
Jeffrey lives in San Francisco by himself, takes public transportation to his job
and does not own a car. He does however frequently drive his girlfriend Joy’s car.
Which of the following policies should Jeffrey purchase to protect himself legally
and how would Joy’s policy respond in the event of an occurrence?
A. Named non-owned coverage policy. Joy’s policy would be secondary Jeffrey’s
would be primary
B. Named non-owned coverage policy. Joy’s liability policy would be primary
Jeffrey’s would be secondary
C. Driver other car endorsement. Joy’s policy would be secondary Jeffrey’s would
be primary
D. Miscellaneous type vehicle endorsement. Joy’s policy would be secondary
Jeffrey’s would be primary
An endorsement to an HO policy form that provides coverage for boats one owns,
rents or borrows is defined as which of the following?
A. Watercraft endorsement
B. Miscellaneous type vehicle endorsement
C. Recreational vehicle endorsement
D. Flotilla liability endorsement
Which of the following HO policy forms offers open peril coverage (A Structure
Coverage) for an insured's home?
A. HO-3
Insuratest 2022
B. HO-4
C. HO-6
D. HO-8
All of the following do not (-) describe a type of loss triggering a claim for
monetary damages due to an insured un-willfully causing injury to another
person or damage to another's property by a negligent act except (-)? Remember
a (-) + a (-) = a positive so you are looking for the term that matches the definition stated
in this question's text.
A. Real property loss
B. Physical loss
C. Personal property loss
D. Liability loss
Which of the following are required of an insured after suffering a collision loss
regardless of fault per a personal auto policies conditions section?
A. Make no attempt to protect the insured property from further loss
B. Allow the insurer to inspect the damaged property before it is repaired
C. Notify the police if damages are in excess of $500
D. Go to the repair shop the claims adjuster recommends
Similar to aircraft and boatowner’s policies which coverages are typically
provided in a boat owner's policy?
A. Flotilla liability
B. Inland Marine, hull, and protection & indemnity
C. Physical damage, liability, and medical payments to driver/operator and
passengers
D. Professional liability, flotilla liability and medical payments to driver/operator and
passengers
Insuratest 2022
Although different in design in terms of coverage what is similar between a
California HO-2, HO-4, and HO-6 policy?
A. Cause of loss coverage A form perils
B. Coverage C policy limits
C. Cause of loss coverage C form perils
D. None of the above
Commercial property theft also called “fidelity coverage” is provided by none (-)
of the following coverage forms except (-)?
A. Commercial property form
B. Building and personal property coverage form
C. Commercial Crime Insurance
D. None of the above coverage forms afford coverage for theft
All of the following are not (-) covered by a Commercial General Liability policy
except (-)?
A. Damage to “Your Work”
B. Damage to “Your Product”
C. Contractual Liability
D. Premises operations
Insuratest 2022
Extra expense insurance affords coverage for all of the following (+) if coverage is
in force at time of loss except (-)?
A. Costs of setting up business in an alternative temporary location, during the
period of restoration
B. Business income including payroll when a business is inoperable after
incurring a loss (Why? This is covered by business income not Extra Expense
coverage).
C. The extra expenses a business incurs to continue a business while damage to
the premises are being restored
D. None of the above
Which of the following perils are covered under the ISO business income
coverage form, choose the best answer?
A. Those recorded on the declarations page
B. Rental income
C. Business Income
D. Ordinary Payroll
California Dreamin landscaping service owner Tom has the choice to purchase
his company's workers compensation from a variety of different insurers
presented by his insurance broker at time of policy renewal. He lives in a state
which has _________________?
A. A statutory state insurance fund
B. A competitive state insurance fund
C. A monopolistic state insurance fund
D. A varietal insurance fund
Which of the following coverages if any come standard with a DP-3 policy that are
not part of a DP-1 policy form?
Insuratest 2022
A. Fair rental value
B. Personal property
C. appurtenant structures
D. Collapse
To terminate a license that is in the possession of an employer which of the
following must occur?
A. The employer need only furnish a notice of employment, automatically
terminating the licensee's license.
B. The licensee must provide written notice of intent to the insurance
commissioner
C. The licensee must give written notice to the employer who must send written
notice to terminate licensee’s license no later than 10 business days following
termination of employment.
D. The license cannot be terminated prior to the expiration date, it becomes inactive
unless the licensee is charged with an insurance related misdemeanor or is convicted of
a felony.
To terminate a licensee’s license that is in the possession of a licensee which of
the following must occur?
A. Nothing, the licensee need only wait until the license expires for non payment of
licensing fees
B. Relinquish all appointments and return books and records to the commissioners
office
C. Deliver the physical document along with notice of relinquishment to the
Commissioner
D. None of the above
Which of the following, if any, will cover a loss that occurs during the policy
period but is discovered 75 days after the expiration date of a claims-made
commercial general liability policy form.
A. Inland marine permit endorsement
Insuratest 2022
B. Supplemental extended reporting period endorsement
C. The loss will not be covered because it was not reported during the policy period
D. Value reporting form endorsement
How would an ISO commercial building and business personal property coverage
form, respond in the event a vandalism loss occurs and adjusters determine the
building has been vacant in excess of 60 straight days at time of loss?
A. The loss is covered 90 consecutive days is the exclusion coverage trigger
B. The adjusted loss payment will incur a coinsurance penalty less the deductible
C. The adjusted loss payment will incur a coinsurance penalty less the deductible
D. The loss is excluded 60 consecutive days is the exclusion coverage trigger
How would an ISO commercial building and business personal property coverage
form, respond in the event a property loss occurs and adjusters determine the
building has been vacant in excess of 60 straight days at time of loss?
A. The loss is covered 90 consecutive days is the exclusion coverage trigger
B. The adjusted loss payment will incur a coinsurance penalty less the deductible
C. The adjusted loss payment will incur a coinsurance penalty less the deductible
D. The adjusted amount of property loss will be reduced by 15%
What is the main purpose of tort law as it pertains to insurance?
A. To properly subrogate insurance claims made
B. To protect an insured from fraudulent suits
C. To prove negligence and determine responsibility for damages
D. To increase the capacity for legal liability claims
Insuratest 2022
Which of the following statements regarding building glass coverage is accurate
with regard to the 2000 ISO editions of the basic, broad, and special causes of
loss forms?
A. Both the basic and broad causes of loss forms provide full coverage for
building glass breakage due to vandalism in the same manner and to the same
extent as damage to other types of covered property
B. Both the broad and special causes of loss forms provide full coverage for building
glass breakage due to vandalism in the same manner and to the same extent as
damage to other types of covered property
C. There is no coverage for building glass breakage due to acts of vandalism unless
the vandalism and malicious mischief endorsement is appended to the policy
D. There is no coverage for building glass breakage due to acts of vandalism
available in either the basic, broad and special forms.
All of the following statements reflect an insurer's use of the inspections and
surveys condition found in a commercial package policy except (-):
A. Recommend changes to mitigate future losses
B. Guarantee safety regulation conformity
C. Rate selection
D. Make a decision on insurability
The cost to replace an insured's damaged property, less depreciation as a means
of indemnification is the formula for?
A. Stated value
B. Replacement cost value
C. Actual cash value
D. Appraised value
An insured submits a fraudulent claim for a workplace injury. Following an
adjuster's investigation, with the aid of worksite camera evidence it is determined
the injury never occurred “at work”. Paid to date on this claim are the following:
Insuratest 2022
Loss of wages $35,000, Chiropractic visits $10,000, physical therapy $15,000.
What is the total fine imposed on the insured for fraudulently submitting a false
claim to the insurer with the intent to deceive? {Remember, payout is 2 X’s the $
amount of the fraudulent claim submitted or $150,000 whichever is greater}
A. $60,000
B. $100,000
C. $120,000
D. $150,000
According to the insurance code no insurer’s rate filing may remain in effect if it
is considered to be any of the following, except (-):
A. Having impartiality
B. Possessing Inequitability
C. Prejudicial in nature
D. Actuarially insufficient
Which of the following best defines “time element ” loss coverage and what are
the two most common coverage form types?
A. How long it takes to repair or replace the damaged property: Business
interruption and third party indemnity
B. How long it takes to repair or replace the damaged property: Seasonal indemnity
and value reporting.
C. How long it takes to repair or replace the damaged property: Business
interruption and Extra Expense
D. Why are you reading this
None of the following statements (-) exemplify the rationale/purpose behind a
commercial package policy inspections and surveys by an insurer except (-)?
Insuratest 2022
A. To correct any OSHA violations
B. To guarantee no losses will occur
C. To determine the rate to be charged
D. To justify a premium increase upon renewal
_____________ damages provide a claimant with the monetary amount necessary
to properly indemnify them, restoring them to the same financial condition that
existed prior to suffering a loss and nothing more while ____________ damages
are designed to punish the negligent party causing said loss?
A. Punitive, General
B. Compensatory, Punitive
C. Special, General
D. General, Special
Damages awarded a personal injury claimant fall into two basic categories
______________ (also known as economic damages) and ______________ (also
known as non-economic damages). Both types of damages are known as
____________ damages or actual damages?
A. Special, General, Compensatory
B. Special, General, Punitive
C. General, Special, Compensatory
D. General, Special, Punitive
A/an _________ performs certain functions normally handled exclusively by
insurers. Responsibilities include binding coverage, underwriting, pricing as well
as appointing agents. They may also settle claims?
A. Underwriter
Insuratest 2022
B. Managing director
C. Agency principal
D. MGA (Managing General Agent)
______________ Is defined as a person (individual or corporation) who negotiates
and binds ceding reinsurance contracts on behalf of an insurer?
A. A credit insurance broker
B. MGA (managing general agent)
C. Insurance agency principal
D. An TPA (Third party administrator)
Motor carriers of goods are responsible for the cargo in their care custody and
control up to a point. The extent or degree of that responsibility is established by
a written contract or through a _____________. While ______________ limits a
common carrier's liability subject to a dollar limitation on the value of the cargo
or the dollar amount of any potential loss?
A. Cargo Policy: inland marine policies
B. Bill of lading: Release Bill of lading
C. Value reporting form: Indemnity agreement
D. Why are you reading this
A personal auto policy conditions “Your Duties After Loss” section states an
insured must notify the police in the event of the following occurrences?
A. Loss caused by theft: Hit and run accidents
B. Loss caused by theft: Accidents in excess of $750 in property damage
C. Loss caused by theft
D. Only need to notify the insurance carrier or agent
Insuratest 2022
What coverage is/are available under the “extended non-owned” coverage
endorsement when added to a personal auto policy to protect an insured who is
furnished with a company car they drive off the clock for personal use as well?
A. Coverage’s (A) Liability, (D) physical damage
B. Coverages (A) Liability medical payments
C. Coverage’s (A) Liability, (B) medical payments, (C) uninsured motorist
D. Liability only
Which of the following peril exclusions contained in the special form
homeowner’s policy contract excludes coverage for mudflow?
A. Flood
B. Earthquake
C. Landslide
D. Subsidence
A representation made by an insured during the application process may be
withdrawn only__________________?
A. any time within the 90 day binder period
B. prior to insurance contract issuance
C. after the effective date
D. before a claim occurs
__________ policies are a blend of__________ and __________ coverage in one
plan and afford property coverage to farm equipment?
A. Transit / standard / substandard
Insuratest 2022
B. Truckers / inland marine / transit
C. Farm / residential / commercial
D. Inland marine / residential / commercial
Which of the following coverages if in force at time of loss would respond to an
insured's car being damaged by a hail storm?
A. Hail Coverage endorsement
B. Collision coverage (D)
C. Comprehensive aka other than collision coverage (D)
D. Miscellaneous type vehicle endorsement
An insured may not purchase a homeowner’s policy form or must have their
current homeowner’s policy form rewritten to a dwelling form for none (-) of the
following reasons except (-)?
A. Their home is for sale and listed on the MLS
B. Their in laws move in to assist with childcare
C. Their home is being renovated
D. The owner no longer resides in the dwelling as it is rented to others
In addition to third party bodily injury and property damage coverage afforded by
coverage (A) all of the following (+) are covered in addition except (-)?
A. Loss of wages if required to attend a hearing at the third party’s insurers request
B. The cost to repair the third parties vehicle up to the in force coverage limit
C. Prejudgment interest supplementary payments
D. Why are you reading this
{ (-) remember the last word wins}
A standard fire policy affords coverage (A) Dwelling and coverage (B)
Other/Appurtenant structures. While a business owners policy covers
(A)____________ and ___________ which includes ____________?
Insuratest 2022
A. Building / Business Personal Property / Tenants improvements and
betterments
B. Building and Personal Property of Others / Care Custody and Control
C. Rental agreement / Business Personal Property / Extra Expense coverage
D. None of the above
_______________ is something that has a tendency to self-destruct and is not
covered (i.e. fruit may spoil during shipment)?
A. Inherent Vice
B. Jettison
C. Barratry
D. None of the above
___________ is to dump cargo overboard in attempts to save a ship from going
under while __________ is/are illegal acts committed by the captain or crew
without the ship owner’s knowledge?
A. Barratry / Jettison
B. Jettison / Barraty
C. Barratry / Embezzlement
D. Jettison / Embezzlement
Which of the following coverage is afforded (+) an insured who purchases a DP3
policy that would otherwise not be covered with the purchase of a DP1?
A. Sprinkler leakage
Insuratest 2022
B. Collapse
C. Additional living expenses
D. All of the above
An insured elects to choose a $500 other than collision deductible and a $1,000
collision deductible to cover his vehicle for physical damage. He is involved in an
accident in which his vehicle suffers an “upset” overturning on highway 17
during a rainstorm. The vehicle suffers $8,000 in total damage. How much will the
insurer pay for the loss?
A. $7,500
B. $7,000
C. $500
D. $1,000
An insured elects to choose a $500 other than collision deductible and a $1,000
collision deductible to cover his vehicle. He is involved in an accident in which
his vehicle suffers an “upset” overturning on highway 17. The vehicle suffers
$8,000 in total damage. How much will the insured pay for the loss?
A. $7,500
B. $7,000
C. $500
D. $1,000
Insuratest 2022
Please get up, walk around,come back in 10-20 min to keep it fresh.
Open-peril(cause of loss) also referred to as special form policies protect an
insured against losses arising from?
A. Perils contained in the exclusions section
B. Perils involving open vacant land
C. All perils, except for those which are specifically excluded (Typically
Earthquake and Flood)
D. Perils specifically named in the policy
Insuratest 2022
Which of the following loss settlement options best describes an option the
insurer may use to indemnity their insureds for property losses based on what
the property in question could have been sold for at the time of loss?
A. Actual cash value
B. Stated value
C. Market value
D. Replacement value
The “examination of commercial books and records” condition found in a
commercial policy states the insurance company possesses the inherent right to
audit an insured's books and records as they relate to their policy during the
policy period or any time within three years after the policy periods end. None of
the following reasons (-) are valid in terms of purpose to exercise this condition
except (-)?
A. To ensure the insureds business practice’s are not unethical
B. To validate the proper amount of premium is charged for the risk
C. To ensure OSHA regulations are not being violated
D. To ensure no material warrants were violated post contract binding
___________ coverage is designed to cover property in transit over land, certain
types of movable property (whether or not ever moved) as well as all risk
coverage for endorsed personal articles such as a wedding rings and precious
medallions?
A. Ocean marine
B. Flotilla liability
C. Inland marine
D. Miscellaneous cargo in transit coverage
An insurance policy may not be transferred to another without:
Insuratest 2022
A. Written consent of the agent
B. Written consent of the insurer
C. Oral consent of the agent
D. Oral consent of the insurer
The “Mexico coverage endorsement” will afford an insured temporary automobile
coverage while visiting Mexico; barring they are in Mexico for no longer than a
maximum of ________ days or less?
A. 7
B. 10
C. 14 days or less
D. 25 days or less
A loss sustained policy form will pay for losses which occurred_________ the
policy period yet are ___________ up to one year after policy _________.
A. during, discovered, renewal
B. prior to, discovered, renewal
C. during, discovered, termination
D. prior to, discovered, termination
Why would an insured Dry cleaning business owner/operator need an inland
marine policy form to cover his clients garments in his possession while being
loaded into owned delivery vans on premises for scheduled delivery?
Insuratest 2022
A. Clients garments in his care custody and control are only covered in the insured's
building location or within 250 ft. from the insured premises
B. Clients garments in his care custody and control are only covered in the
insured's building location or within 100 ft. from the insured premises
C. An insured dry cleaner will need only pollution liability inland marine insurance is
covered automatically by this policy form
D. Nne of the above
What two sub-coverage lines are listed on a Commercial General liability policies
declarations page (D.I.C.E)?
A. Premises and operations and Products and completed operations
B. Rented to you sub limit and professional liability coverage
C. Medical payments to others and rented to you sub limit
D. Products and completed operations and Medical payments to others
Which of the following best describes Premises and operations one of the sub-
coverages on a Commercial General liability policy's declarations page (D.I.C.E)?
A. Covers damage caused by “your work” exclusively
B. Covers losses at the location of the business
C. Cover losses that occur away from the business
D. None of the above
Which of the following best describes Products Completed operations one of the
sub-coverages on a Commercial General liability policy's declarations page
(D.I.C.E)?
A. Cover losses that occur away from the business
B. Covers losses at the location of the business
C. Covers damage caused by “your work” exclusively
Insuratest 2022
D. None of the above
Which of the following is/are not covered (-) by a Commercial General liability
policy?
A. Aircrafts
B. Alcoholic Business
C. Host liquor liability
D. Both A and B
Which of the following is/are (+) covered by a Commercial General liability policy?
A. Aircrafts
B. Alcoholic Business
C. Host liquor liability
D. Professional liability
Truckers/motor carrier’s coverage minimum standard liability limits are?
A. $ 750,000
B. $ 1,000,000
C. $ 3,000,000
D. $ 5,000,000
Truckers/motor carrier’s coverage minimum certain hazardous materials liability
limits are?
A. $ 750,000
B. $ 1,000,000
C. $ 3,000,000
D. $ 5,000,000
Insuratest 2022
Truckers/motor carrier’s coverage minimum explosive materials liability limits
are?
A. $ 750,000
B. $ 1,000,000
C. $ 3,000,000
D. $ 5,000,000
None of the following (-) coverages represent the principal difference between an
HO-3 and an HO-5 policy except (-)?
A. Coverage A and B causes of loss (perils)
B. Coverage C policy limits
C. Coverage C causes of loss (perils)
D. Coverage D policy limits
None of the following (-) coverages will protect physicians, surgeons, and
dentists for their professional activities except (-)?
A. Commercial general liability
B. Medical Malpractice coverage
C. Professional liability
D. Errors and omissions coverage
Which of the following terms names the process through which a mutual insurer
becomes a stock insurer sometimes called stocking or privatization?
A. Mutualization
B. Reorganization
C. Stock Split
D. Demutualization
Insuratest 2022
The “Mexico coverage endorsement” will afford an insured temporary automobile
coverage while visiting Mexico; barring they are in Mexico for no longer than a
maximum of 10 days or less and do not drive more than ________ miles past the
Mexico/U.S. border?
A. 25
B. 50
C. 75
D. 100
An agent submits an application to an insurer he is not yet appointed with on
behalf of an insured. The insurer accepts the application and issues the policy.
What must the insurer now provide the insurance commissioner and in what time
frame?
A. An agent by no means may submit an application to a carrier not yet
appointed with
B. Send to the Insurance Commissioner a written notice of agent’s
appointment within 10 days of receipt of said application
C. Send to the Insurance Commissioner a written notice of agent’s
appointment within 14 days of receipt of said application
D. Send to the Insurance Commissioner a written notice of agent’s
appointment within 30 days of receipt of said application
Gurevich and Batstone agency represents Infinity Insurance Corporation. They
may “express” the relationship between the two entities in all of the following (+)
ways except (-)?
A. Gurevich and Batstone agency underwriting for Infinity Insurance
Corporation
B. Gurevich and Batstone agency proudly representing Infinity Insurance
Corporation
C. Gurevich and Batstone agency dedicated sales associates for Infinity Insurance
Corporation
D. Why are you reading this
Common policy conditions (D.I.C.E) include none of the following (-), except (-)?
Insuratest 2022
A. Cancellation provisions/notices and who may make policy changes
B. Examination of the insured's books and records and Insurer inspections
and surveys
C. Premiums charged for risk transference (insurance) as well as transfer of
rights/assignment
D. All of the above
Which of the following per-day transportation expense i.e. rental car coverage
limits are afforded an insured who has an unendorsed personal auto policy in
force at time of loss?
A. $20
B. $25
C. $35
D. None
Of an MGA (Managing General Agent) all of the following are true (+) except (-)?
A. Has the power to appoint, supervise and terminate agent-broker appointment
contracts
B. Has the express capability to either accept or decline risks based on actuarial
principles
C. Possesses a valid fire and casualty agent-broker’s license
D. May not assist with the claims handling process
Part 1 of a Workers Compensation Policy protects against statutory claims and
it’s benefits include (+)?
A. Medical with no time limit and no dollar limit as well as rehabilitation
benefits
Insuratest 2022
B. Disability Income at 66 ⅔% of a worker’s salary
C. Death benefits in California: lump sum of up to $5,000 and income to the
surviving spouse
D. All of the above
Part 2 of a Worker’s Compensation Policy protects against Common Law Claims
and it’s benefits include (+)?
A. Covers people with claims outside the scope of workers compensation part
1, with an unlimited lawsuit potential
B. Subsequent/Second Injury Fund
C. Pays the increase in compensation from a previous injury and or a
secondary or subsequent injury
D. All of the above
Of the following loss exposures which one encompasses losses that of which
may occur on an insured's land or in“real property” built on said land?
A. Liability loss exposure
B. Personnel loss exposure
C. Farm liability
D. Real property liability endorsement
{OK here comes three sneaky questions…}
Xavier is involved in a parking lot accident with Yoenis. Xavier’s vehicle suffers
$6,000 in damage but adjusters determine that Xavier is 50% responsible for the
accident. Xavier has a $1,000 collision and a $500 other than collision deductible
How much would Xavier receive under comparative negligence rules?
A. $ 0
Insuratest 2022
B. $ 2,000
C. $ 4,500
D. $ 5,000
Xavier is involved in a parking lot accident with Yoenis. Xavier’s vehicle suffers
$6,000 in damage but adjusters determine that Xavier is 49% responsible for the
accident. Xavier has a $1,000 collision and a $500 other than collision deductible
How much would Xavier receive under contributory negligence rules?
A. $ 0
B. $ 2,000
C. $ 4,500
D. $ 5,000
Xavier is involved in a parking lot accident with Yoenis. Xavier’s vehicle suffers
$6,000 in damage but adjusters determine that Xavier is 51% responsible for the
accident. Xavier has a $1,000 collision and a $500 other than collision deductible
How much would Xavier receive under contributory negligence rules?
A. $ 0
B. $ 2,000
C. $ 4,500
D. $ 5,000
A flatbed truck with a gross vehicle weight of 9,500 lbs used to haul hay on
Schweig ranch may be eligible for a personal policy because it is used in the
business of:
A. Commercial transport
B. Hauling cargo for a fee
C. Farming or ranching
Insuratest 2022
D. Hauling agricultural goods for a fee
Which of the following is true (+) of a Surplus lines broker?
A. An Surplus lines broker represents non-admitted companies
B. Must post a $50,000 bond
C. Must be appointed and may have binding authority.
D. Both A and B
All of the following statements regarding the term “concealment” with respects to
an insured's application are true (+) except (-)?
A. Concealment of material relevant to the formulation of a contract once
discovered are grounds for automatic declination by an insurer
B. Concealment does not occur if the insured was not asked about a
particular subject. i.e. no inquiry was made as to the fact that their building is
located next to an explosives factory
C. Concealment may be both intentional unintentional
D. None of the above
{Remember concealment is the intentional withholding of material information by an
insured during the formulation of a contract. Although concealment is relevant to
property and casualty insurance, it is most often associated with Life insurance
contracts. If misrepresentation is discovered at any time during the policy period it may
give the insurer grounds to rescind/flat rate cancel a policy as though it never existed.
Example: an insured conceals the fact that he is a part time skydiving instructor on
weekends and dies in a skydiving accident six months into a life insurance policy
contract. The insurance company would most likely rescind the life insurance policy
Insuratest 2022
purchased and return the premiums paid to the insured's beneficiary and not pay the
death benefit on the grounds of concealment.}
An insured owns a commercial warehouse building insured by an unendorsed
commercial package policy. The building was vacant for a period of 75
consecutive days before a loss occurred. The policy will cover a property loss if
caused by all of the perils listed below except (-):
A. Vandalism and plate glass breakage
B. Fire
C. Windstorm
D. Lightning
Which of the following types of in force policies will pay for either a bodily injury
or property damage loss that occurs, or is discovered during the policy period?
A. Pollution liability policy
B. Auto liability policy
C. Claims made commercial liability policy
D. Occurrence general liability policy
What is the cut-off time period regarding claims submitted by an insured during
the policy period under the unendorsed claims-made commercial general liability
policy coverage form?
A. 30 days
B. 60 days
C. 90 days
D. 6 months
An insured borrows his neighbor's truck to pick up a Christmas Tree how do the
insureds and neighbors policies respond in the event an auto accident takes
place?
Insuratest 2022
A. Only the neighbor's policy provides coverage; there is no coverage by the driver's
policy
B. The neighbor's policy is secondary and the driver's policy is primary. Primary
coverage follows the driver
C. The neighbor's policy is primary and the driver's policy is excess. Primary
coverage follows the car
D. Not enough information regarding accident and or proof of negligence
The following loss descriptions are all indicative of "ideally insurable loss
exposure," except (-):
A. Losses that are accidental
B. Losses that are definite and measurable
C. Large number of exposure units
D. Losses that are catastrophic
Vacancy conditions under a basic commercial property policy are waived for
specified perils during the vacancy permit period barring the ___________ is
added to a building and personal property coverage form at time of loss.
A. Special form perils endorsement
B. Extended form perils endorsement
C. Vacancy permit endorsement
D. Vandalism acts indemnity clause
The commercial building and personal property coverage affords coverage for
“your business personal property.” “Your” as a coverage definition references
none (-) of the following except (-)?
A. Only the first named insured listed in the policy declarations page
B. Only the named insured(s) shown in the declarations
Insuratest 2022
C. Owners of property in the care custody and control of the first named insured
D. none of the above
The process whereby insurers evaluate risks, decide which customers to insure,
and what coverage(s) to offer; all while determining the risk transference rate to
charge, based on sound actuarial principles is called?
A. The law of large numbers
B. Underwriting
C. Rate Factoring
D. Adverse Selection
The liability coverage specific to a yacht owners policy is called?
A. Slip liability
B. Personal injury protection
C. Protection and indemnity
D. Professional liability
An insured is in the business of pyrotechnic entertainment and is required to
furnish proof of liability coverage prior to being booked for the State Fair’s 4th of
july fireworks display. Since this risk is out of the ordinary and inherently
dangerous in nature. Where would the insured go in order to obtain this type of
liability coverage? Choose the best answer:
A. Pyrotechnician labor union
B. From an excess and or surplus lines broker
C. From a nonadmitted insurer
D. From an alien insurer
Coverage afforded by the commercial equipment dealer’s property coverage form
would include all of the the following (+), except (-)?
A. Customer’s equipment in insured's care custody and control while in for service
B. New equipment for sale
Insuratest 2022
C. Motorized vehicles CHP approved for use on state highways
D. Lease returned equipment for resale
Of the following loss scenarios which of the following is not covered (-) by
coverage (D) collision and other than collision coverage of an insured's personal
auto policy?
A. Tree limb falls in a severe storm damaging an insured vehicle’s hood
B. Christmas presents left in the backseat of a vehicle overnight in a parking
garage are stolen
C. Vehicle suffers front end damage during an earthquake
D. Vehicle suffers water damage during a severe rain storm
What is the formula for net income?
A. Total net loss minus expenses
B. Total net gain less taxes and fees
C. Total revenue minus total expenses
D. Total revenue plus total payroll
A standard ISO HO-3 coverage form also known as the “special form” policy
affords coverage to an insured's real property on a/an __________ basis while it’s
personal property is covered on a/an ____________ basis.
A. All risk, broad form
B. All risk, open peril
C. All risk, special form
D. Special form, open peril
Insuratest 2022
Cargo coverage extends to none of the following (-) except (-)?
A. Products being shipped
B. covers the freight and reimburses the shipping charges in the event of a
covered loss
C. Inherent Vice
D. Both A and B
All of the following types of businesses are eligible for their business personal
property aka “stock” to be covered under an inland marine commercial property
coverage form except (-)?
A. Stringed instrument repair shop
B. Grocery Store
C. Dry cleaners
D. Jewelry store
After running an annual DMV check and finding two major violations on the Cargo
coverage extends to none of the following (-) except (-)?
A. Products being shipped
B. covers the freight and reimburses the shipping charges in the event of a
covered loss
C. Inherent Vice
D. Both A and B
Cargo coverage extends to none of the following (-) except (-)?
A. Products being shipped
B. covers the freight and reimburses the shipping charges in the event of a
covered loss
C. Inherent Vice
Insuratest 2022
D. Both A and B
An insured purchases a new vehicle and contacts his agent the next day to add
the vehicle to his existing auto policy contract. What temporary coverage is
extended from his existing auto policy during the time it takes to bind coverage?
A. No coverage available must purchases temporary liability coverage from the
dealer before driving off the lot
B. The lowest coverage limits for any scheduled on his declarations page in force at
the time he took ownership of the new vehicle.
C. The broadest coverage for any scheduled on his declarations page in force
at the time he took ownership of the new vehicle.
D. Temporary coverage for liability only is afforded by his in force policy’s insuring
agreement
Which of the following homeowners coverage forms is most appropriate for a
custom home with high end furniture and a state of the art sound-entertainment
system inside?
A. HO-3
B. HO-4
C. HO-5
D. HO-8
In addition to the mandatory online or classroom pre-license training program
what else must an applicant complete whether it is for a life agent license,
personal lines agent license or a fire and casualty license prior to applying for an
actual state license?
A. A course on ethical marketing and sales techniques
B. A four hour course on ethical presentation standards
C. None of the above
Insuratest 2022
D. A 12 hour course on theinsur ance code and ethics
When determining the value of a commercial property to estimate the 80%
minimum coinsurance limit which of the following cost measures is implemented
by the insurer?
A. Replacement cost value
B. Fair market value using comps as a value estimator
C. The actual dollar amount to replace said commercial property at today’s
construction labor rates & material costs
D. The cost to replace said property less depreciation and a coinsurance trigger
surcharge
{You may see both variations of this question on your exam, both answers are correct
as a “Tort is a legal wrong other than a crime or a breach of contract”}
On the basis of tort law one may be held legally liable for a claim if any of the
following are proven except (-)?
A. Negligence
B. Legal wrong
C. Absolute liability
D. Breach of contract
On the basis of tort law one may be held legally liable for a claim if any of the
following are proven except (-)?
A. Negligence
B. Crime
C. Legal wrong
D. Absolute liability
Insuratest 2022
Alvin is involved an accident with Bernice. Alvins vehicle incurs $20,000 in
physical damage. Adjusters determine Alvin is 25% responsible for the accident.
How much would Alvin receive under comparative negligence rules?
A. $ 0
B. $ 10,000
C. $ 15,00
D. $ 20,000
__________ is defined as the process an insurer undertakes to calculate an
insured's risk transference premium based on sound actuarial principles?
A. Rate-making
B. Premium calculating
C. Underwriting
D. Loss assessment
The unendorsed personal auto policy territory limits include none of the following
(-) except (-): {Remember! (-) + (-) = + “What’s included”}
A. Cayman Islands
B. Toronto, Canada
C. Tijuana, Mexico
D. Cuba
Which of the following losses caused by the negligent actions of an insured
would result in a third party claim award for physical injury and/or property
damage?
A. Physical property loss
B. Personnel property loss
C. Liability loss
D. Arbitration loss
Insuratest 2022
Which of the following is true about the (CGL) Commercial General Liability
coverage limit referenced on the declarations page?
A. The CGL limit includes the products completed operations aggregate limit and
advertising injury limit
B. The CGL limit is a separate limit from the products completed operations
aggregate limit and advertising injury limit
C. The CGL limit is another name for per occurrence limit.
D. The CGL limit is another name for the aggregate limit
According to insurance code active agents and brokers must have their books
and records made available to the commissioner’s office for examination
______________?
A. Once every two years on the month of the agent or broker's license renewal
B. Once a year on the month of the agent or broker's license renewal
C. Within ten days after receiving a certified letter from the commissioner's office
requesting said examination
D. At all times and as often as the commissioner in his or her discretion
deems appropriate
How often may the insurance commissioner's office conduct an examination of
the books and records of an active agent or broker per the Insurance code?
A. Not less frequently than once every five years
B. Reviews must be done annually
C. Only in the event of a consumer complaint as often as necessary
D. Once every three years
Which of the following terms best describes the claim settlement option whereby
an insured will replace an insured's lost, damaged or stolen property with that of
the same like kind, quality, make and or construction?
Insuratest P&C Exam Mastery System .docx
Insuratest P&C Exam Mastery System .docx
Insuratest P&C Exam Mastery System .docx
Insuratest P&C Exam Mastery System .docx
Insuratest P&C Exam Mastery System .docx
Insuratest P&C Exam Mastery System .docx
Insuratest P&C Exam Mastery System .docx
Insuratest P&C Exam Mastery System .docx
Insuratest P&C Exam Mastery System .docx
Insuratest P&C Exam Mastery System .docx
Insuratest P&C Exam Mastery System .docx
Insuratest P&C Exam Mastery System .docx
Insuratest P&C Exam Mastery System .docx
Insuratest P&C Exam Mastery System .docx
Insuratest P&C Exam Mastery System .docx
Insuratest P&C Exam Mastery System .docx
Insuratest P&C Exam Mastery System .docx
Insuratest P&C Exam Mastery System .docx
Insuratest P&C Exam Mastery System .docx
Insuratest P&C Exam Mastery System .docx
Insuratest P&C Exam Mastery System .docx
Insuratest P&C Exam Mastery System .docx
Insuratest P&C Exam Mastery System .docx
Insuratest P&C Exam Mastery System .docx
Insuratest P&C Exam Mastery System .docx
Insuratest P&C Exam Mastery System .docx
Insuratest P&C Exam Mastery System .docx
Insuratest P&C Exam Mastery System .docx
Insuratest P&C Exam Mastery System .docx
Insuratest P&C Exam Mastery System .docx
Insuratest P&C Exam Mastery System .docx
Insuratest P&C Exam Mastery System .docx
Insuratest P&C Exam Mastery System .docx
Insuratest P&C Exam Mastery System .docx
Insuratest P&C Exam Mastery System .docx
Insuratest P&C Exam Mastery System .docx
Insuratest P&C Exam Mastery System .docx
Insuratest P&C Exam Mastery System .docx
Insuratest P&C Exam Mastery System .docx
Insuratest P&C Exam Mastery System .docx
Insuratest P&C Exam Mastery System .docx
Insuratest P&C Exam Mastery System .docx
Insuratest P&C Exam Mastery System .docx
Insuratest P&C Exam Mastery System .docx
Insuratest P&C Exam Mastery System .docx
Insuratest P&C Exam Mastery System .docx
Insuratest P&C Exam Mastery System .docx
Insuratest P&C Exam Mastery System .docx
Insuratest P&C Exam Mastery System .docx
Insuratest P&C Exam Mastery System .docx
Insuratest P&C Exam Mastery System .docx
Insuratest P&C Exam Mastery System .docx
Insuratest P&C Exam Mastery System .docx
Insuratest P&C Exam Mastery System .docx
Insuratest P&C Exam Mastery System .docx
Insuratest P&C Exam Mastery System .docx
Insuratest P&C Exam Mastery System .docx
Insuratest P&C Exam Mastery System .docx
Insuratest P&C Exam Mastery System .docx
Insuratest P&C Exam Mastery System .docx
Insuratest P&C Exam Mastery System .docx
Insuratest P&C Exam Mastery System .docx
Insuratest P&C Exam Mastery System .docx
Insuratest P&C Exam Mastery System .docx
Insuratest P&C Exam Mastery System .docx
Insuratest P&C Exam Mastery System .docx
Insuratest P&C Exam Mastery System .docx
Insuratest P&C Exam Mastery System .docx
Insuratest P&C Exam Mastery System .docx
Insuratest P&C Exam Mastery System .docx
Insuratest P&C Exam Mastery System .docx
Insuratest P&C Exam Mastery System .docx
Insuratest P&C Exam Mastery System .docx
Insuratest P&C Exam Mastery System .docx
Insuratest P&C Exam Mastery System .docx
Insuratest P&C Exam Mastery System .docx
Insuratest P&C Exam Mastery System .docx
Insuratest P&C Exam Mastery System .docx
Insuratest P&C Exam Mastery System .docx
Insuratest P&C Exam Mastery System .docx
Insuratest P&C Exam Mastery System .docx
Insuratest P&C Exam Mastery System .docx
Insuratest P&C Exam Mastery System .docx
Insuratest P&C Exam Mastery System .docx
Insuratest P&C Exam Mastery System .docx
Insuratest P&C Exam Mastery System .docx
Insuratest P&C Exam Mastery System .docx
Insuratest P&C Exam Mastery System .docx
Insuratest P&C Exam Mastery System .docx
Insuratest P&C Exam Mastery System .docx
Insuratest P&C Exam Mastery System .docx
Insuratest P&C Exam Mastery System .docx
Insuratest P&C Exam Mastery System .docx
Insuratest P&C Exam Mastery System .docx
Insuratest P&C Exam Mastery System .docx
Insuratest P&C Exam Mastery System .docx
Insuratest P&C Exam Mastery System .docx
Insuratest P&C Exam Mastery System .docx
Insuratest P&C Exam Mastery System .docx
Insuratest P&C Exam Mastery System .docx
Insuratest P&C Exam Mastery System .docx
Insuratest P&C Exam Mastery System .docx
Insuratest P&C Exam Mastery System .docx
Insuratest P&C Exam Mastery System .docx
Insuratest P&C Exam Mastery System .docx
Insuratest P&C Exam Mastery System .docx
Insuratest P&C Exam Mastery System .docx
Insuratest P&C Exam Mastery System .docx
Insuratest P&C Exam Mastery System .docx
Insuratest P&C Exam Mastery System .docx
Insuratest P&C Exam Mastery System .docx
Insuratest P&C Exam Mastery System .docx
Insuratest P&C Exam Mastery System .docx
Insuratest P&C Exam Mastery System .docx
Insuratest P&C Exam Mastery System .docx
Insuratest P&C Exam Mastery System .docx
Insuratest P&C Exam Mastery System .docx
Insuratest P&C Exam Mastery System .docx
Insuratest P&C Exam Mastery System .docx
Insuratest P&C Exam Mastery System .docx
Insuratest P&C Exam Mastery System .docx

More Related Content

Similar to Insuratest P&C Exam Mastery System .docx

Making the grade_exam_tips_from_doctor_wellgood[1]
Making the grade_exam_tips_from_doctor_wellgood[1]Making the grade_exam_tips_from_doctor_wellgood[1]
Making the grade_exam_tips_from_doctor_wellgood[1]Karen O'Byrne
 
Creating the learning zone
Creating the learning zoneCreating the learning zone
Creating the learning zoneJames Sadler
 
Be successful in examination dr. shriniwas kashalikar
Be successful in examination dr. shriniwas kashalikarBe successful in examination dr. shriniwas kashalikar
Be successful in examination dr. shriniwas kashalikarshriniwas kashalikar
 
Be successful in examination dr. shriniwas kashalikar
Be successful in examination dr. shriniwas kashalikarBe successful in examination dr. shriniwas kashalikar
Be successful in examination dr. shriniwas kashalikarshriniwas kashalikar
 
Be successful in examination dr. shriniwas kashalikar
Be successful in examination dr. shriniwas kashalikarBe successful in examination dr. shriniwas kashalikar
Be successful in examination dr. shriniwas kashalikarshriniwas kashalikar
 
Test Taking Strategies
Test Taking StrategiesTest Taking Strategies
Test Taking Strategiescpnators
 
Breaking down an exam question
Breaking down an exam questionBreaking down an exam question
Breaking down an exam questionNatalieKunovic
 
BCS written exam preparation and BCS Preliminary exam preparation
BCS written exam preparation and BCS Preliminary exam preparationBCS written exam preparation and BCS Preliminary exam preparation
BCS written exam preparation and BCS Preliminary exam preparationZahid Hasan
 
ManagingTestAnxiety
ManagingTestAnxietyManagingTestAnxiety
ManagingTestAnxietymcneeteach
 
Assessment is not about the teacher; it’s about the student.2L.docx
Assessment is not about the teacher; it’s about the student.2L.docxAssessment is not about the teacher; it’s about the student.2L.docx
Assessment is not about the teacher; it’s about the student.2L.docxfredharris32
 
Scat Tutoring by Top Grader
Scat Tutoring by Top GraderScat Tutoring by Top Grader
Scat Tutoring by Top GraderVarsha Kirnapure
 

Similar to Insuratest P&C Exam Mastery System .docx (20)

Testtakingskills6 15-12
Testtakingskills6 15-12Testtakingskills6 15-12
Testtakingskills6 15-12
 
Testtakingskills5 24-12
Testtakingskills5 24-12Testtakingskills5 24-12
Testtakingskills5 24-12
 
Making the grade_exam_tips_from_doctor_wellgood[1]
Making the grade_exam_tips_from_doctor_wellgood[1]Making the grade_exam_tips_from_doctor_wellgood[1]
Making the grade_exam_tips_from_doctor_wellgood[1]
 
Creating the learning zone
Creating the learning zoneCreating the learning zone
Creating the learning zone
 
Be successful in examination dr. shriniwas kashalikar
Be successful in examination dr. shriniwas kashalikarBe successful in examination dr. shriniwas kashalikar
Be successful in examination dr. shriniwas kashalikar
 
Be successful in examination dr. shriniwas kashalikar
Be successful in examination dr. shriniwas kashalikarBe successful in examination dr. shriniwas kashalikar
Be successful in examination dr. shriniwas kashalikar
 
Be successful in examination dr. shriniwas kashalikar
Be successful in examination dr. shriniwas kashalikarBe successful in examination dr. shriniwas kashalikar
Be successful in examination dr. shriniwas kashalikar
 
Tips and tricks for the pmp exam
Tips and tricks for the pmp examTips and tricks for the pmp exam
Tips and tricks for the pmp exam
 
Jeopardy Test
Jeopardy TestJeopardy Test
Jeopardy Test
 
1
11
1
 
Exam and Test Preparation
Exam and Test PreparationExam and Test Preparation
Exam and Test Preparation
 
Psych test tips
Psych test tipsPsych test tips
Psych test tips
 
Test Taking Strategies
Test Taking StrategiesTest Taking Strategies
Test Taking Strategies
 
Breaking down an exam question
Breaking down an exam questionBreaking down an exam question
Breaking down an exam question
 
BCS written exam preparation and BCS Preliminary exam preparation
BCS written exam preparation and BCS Preliminary exam preparationBCS written exam preparation and BCS Preliminary exam preparation
BCS written exam preparation and BCS Preliminary exam preparation
 
How to Score More in Exams.pdf
How to Score More in Exams.pdfHow to Score More in Exams.pdf
How to Score More in Exams.pdf
 
ManagingTestAnxiety
ManagingTestAnxietyManagingTestAnxiety
ManagingTestAnxiety
 
How to handle nervousness during interview
How to handle nervousness during interviewHow to handle nervousness during interview
How to handle nervousness during interview
 
Assessment is not about the teacher; it’s about the student.2L.docx
Assessment is not about the teacher; it’s about the student.2L.docxAssessment is not about the teacher; it’s about the student.2L.docx
Assessment is not about the teacher; it’s about the student.2L.docx
 
Scat Tutoring by Top Grader
Scat Tutoring by Top GraderScat Tutoring by Top Grader
Scat Tutoring by Top Grader
 

More from Daniel J. Mastrantonio

Study lounge e book ca P&C exam mastery system as of 7.7.16
Study lounge e book ca P&C exam mastery system as of 7.7.16Study lounge e book ca P&C exam mastery system as of 7.7.16
Study lounge e book ca P&C exam mastery system as of 7.7.16Daniel J. Mastrantonio
 
The Complete Study Lounge Sessions Non CA Specific
The Complete Study Lounge Sessions Non CA SpecificThe Complete Study Lounge Sessions Non CA Specific
The Complete Study Lounge Sessions Non CA SpecificDaniel J. Mastrantonio
 
Revised ABC's and 123's Part 1 Non CA Specific
Revised ABC's and 123's Part 1 Non CA SpecificRevised ABC's and 123's Part 1 Non CA Specific
Revised ABC's and 123's Part 1 Non CA SpecificDaniel J. Mastrantonio
 
Revised ABC's and 123's Part 2 as of 6.18
Revised ABC's and 123's Part 2 as of 6.18Revised ABC's and 123's Part 2 as of 6.18
Revised ABC's and 123's Part 2 as of 6.18Daniel J. Mastrantonio
 
Revised ABC's and 123's Part 1 as of 6.18
Revised ABC's and 123's Part 1 as of 6.18Revised ABC's and 123's Part 1 as of 6.18
Revised ABC's and 123's Part 1 as of 6.18Daniel J. Mastrantonio
 

More from Daniel J. Mastrantonio (18)

USG All States Part Four.pdf
USG All States Part Four.pdfUSG All States Part Four.pdf
USG All States Part Four.pdf
 
USG All States Part Three.pdf
USG All States Part Three.pdfUSG All States Part Three.pdf
USG All States Part Three.pdf
 
USG All States Part Two.pdf
USG All States Part Two.pdfUSG All States Part Two.pdf
USG All States Part Two.pdf
 
USG All States Part One.pdf
USG All States Part One.pdfUSG All States Part One.pdf
USG All States Part One.pdf
 
Study lounge e book ca P&C exam mastery system as of 7.7.16
Study lounge e book ca P&C exam mastery system as of 7.7.16Study lounge e book ca P&C exam mastery system as of 7.7.16
Study lounge e book ca P&C exam mastery system as of 7.7.16
 
The Complete Study Lounge Sessions Non CA Specific
The Complete Study Lounge Sessions Non CA SpecificThe Complete Study Lounge Sessions Non CA Specific
The Complete Study Lounge Sessions Non CA Specific
 
USG Part 4 v2 Non CA Specific
USG Part 4 v2 Non CA SpecificUSG Part 4 v2 Non CA Specific
USG Part 4 v2 Non CA Specific
 
USG Part 3 v2 Non CA Specific
USG Part 3 v2 Non CA SpecificUSG Part 3 v2 Non CA Specific
USG Part 3 v2 Non CA Specific
 
USG Part 2 v2 Non CA Specific
USG Part 2 v2 Non CA SpecificUSG Part 2 v2 Non CA Specific
USG Part 2 v2 Non CA Specific
 
USG Part 1 v2 Non CA Specific
USG Part 1 v2 Non CA SpecificUSG Part 1 v2 Non CA Specific
USG Part 1 v2 Non CA Specific
 
Revised ABC's and 123's Part 1 Non CA Specific
Revised ABC's and 123's Part 1 Non CA SpecificRevised ABC's and 123's Part 1 Non CA Specific
Revised ABC's and 123's Part 1 Non CA Specific
 
Revised ABC's and 123's Part 2 as of 6.18
Revised ABC's and 123's Part 2 as of 6.18Revised ABC's and 123's Part 2 as of 6.18
Revised ABC's and 123's Part 2 as of 6.18
 
Revised ABC's and 123's Part 1 as of 6.18
Revised ABC's and 123's Part 1 as of 6.18Revised ABC's and 123's Part 1 as of 6.18
Revised ABC's and 123's Part 1 as of 6.18
 
The Study Lounge E-Book
The Study Lounge E-BookThe Study Lounge E-Book
The Study Lounge E-Book
 
USG Part 4 v2
USG Part 4 v2USG Part 4 v2
USG Part 4 v2
 
USG Part 3 v2
USG Part 3 v2USG Part 3 v2
USG Part 3 v2
 
USG Part 2 v2
USG Part 2 v2USG Part 2 v2
USG Part 2 v2
 
USG Part 1 v2
USG Part 1 v2USG Part 1 v2
USG Part 1 v2
 

Recently uploaded

SOCIAL AND HISTORICAL CONTEXT - LFTVD.pptx
SOCIAL AND HISTORICAL CONTEXT - LFTVD.pptxSOCIAL AND HISTORICAL CONTEXT - LFTVD.pptx
SOCIAL AND HISTORICAL CONTEXT - LFTVD.pptxiammrhaywood
 
Paris 2024 Olympic Geographies - an activity
Paris 2024 Olympic Geographies - an activityParis 2024 Olympic Geographies - an activity
Paris 2024 Olympic Geographies - an activityGeoBlogs
 
Organic Name Reactions for the students and aspirants of Chemistry12th.pptx
Organic Name Reactions  for the students and aspirants of Chemistry12th.pptxOrganic Name Reactions  for the students and aspirants of Chemistry12th.pptx
Organic Name Reactions for the students and aspirants of Chemistry12th.pptxVS Mahajan Coaching Centre
 
CARE OF CHILD IN INCUBATOR..........pptx
CARE OF CHILD IN INCUBATOR..........pptxCARE OF CHILD IN INCUBATOR..........pptx
CARE OF CHILD IN INCUBATOR..........pptxGaneshChakor2
 
Mastering the Unannounced Regulatory Inspection
Mastering the Unannounced Regulatory InspectionMastering the Unannounced Regulatory Inspection
Mastering the Unannounced Regulatory InspectionSafetyChain Software
 
Concept of Vouching. B.Com(Hons) /B.Compdf
Concept of Vouching. B.Com(Hons) /B.CompdfConcept of Vouching. B.Com(Hons) /B.Compdf
Concept of Vouching. B.Com(Hons) /B.CompdfUmakantAnnand
 
Presiding Officer Training module 2024 lok sabha elections
Presiding Officer Training module 2024 lok sabha electionsPresiding Officer Training module 2024 lok sabha elections
Presiding Officer Training module 2024 lok sabha electionsanshu789521
 
How to Make a Pirate ship Primary Education.pptx
How to Make a Pirate ship Primary Education.pptxHow to Make a Pirate ship Primary Education.pptx
How to Make a Pirate ship Primary Education.pptxmanuelaromero2013
 
The Most Excellent Way | 1 Corinthians 13
The Most Excellent Way | 1 Corinthians 13The Most Excellent Way | 1 Corinthians 13
The Most Excellent Way | 1 Corinthians 13Steve Thomason
 
A Critique of the Proposed National Education Policy Reform
A Critique of the Proposed National Education Policy ReformA Critique of the Proposed National Education Policy Reform
A Critique of the Proposed National Education Policy ReformChameera Dedduwage
 
Kisan Call Centre - To harness potential of ICT in Agriculture by answer farm...
Kisan Call Centre - To harness potential of ICT in Agriculture by answer farm...Kisan Call Centre - To harness potential of ICT in Agriculture by answer farm...
Kisan Call Centre - To harness potential of ICT in Agriculture by answer farm...Krashi Coaching
 
Science 7 - LAND and SEA BREEZE and its Characteristics
Science 7 - LAND and SEA BREEZE and its CharacteristicsScience 7 - LAND and SEA BREEZE and its Characteristics
Science 7 - LAND and SEA BREEZE and its CharacteristicsKarinaGenton
 
mini mental status format.docx
mini    mental       status     format.docxmini    mental       status     format.docx
mini mental status format.docxPoojaSen20
 
Software Engineering Methodologies (overview)
Software Engineering Methodologies (overview)Software Engineering Methodologies (overview)
Software Engineering Methodologies (overview)eniolaolutunde
 
Alper Gobel In Media Res Media Component
Alper Gobel In Media Res Media ComponentAlper Gobel In Media Res Media Component
Alper Gobel In Media Res Media ComponentInMediaRes1
 
Industrial Policy - 1948, 1956, 1973, 1977, 1980, 1991
Industrial Policy - 1948, 1956, 1973, 1977, 1980, 1991Industrial Policy - 1948, 1956, 1973, 1977, 1980, 1991
Industrial Policy - 1948, 1956, 1973, 1977, 1980, 1991RKavithamani
 
Separation of Lanthanides/ Lanthanides and Actinides
Separation of Lanthanides/ Lanthanides and ActinidesSeparation of Lanthanides/ Lanthanides and Actinides
Separation of Lanthanides/ Lanthanides and ActinidesFatimaKhan178732
 
18-04-UA_REPORT_MEDIALITERAСY_INDEX-DM_23-1-final-eng.pdf
18-04-UA_REPORT_MEDIALITERAСY_INDEX-DM_23-1-final-eng.pdf18-04-UA_REPORT_MEDIALITERAСY_INDEX-DM_23-1-final-eng.pdf
18-04-UA_REPORT_MEDIALITERAСY_INDEX-DM_23-1-final-eng.pdfssuser54595a
 

Recently uploaded (20)

SOCIAL AND HISTORICAL CONTEXT - LFTVD.pptx
SOCIAL AND HISTORICAL CONTEXT - LFTVD.pptxSOCIAL AND HISTORICAL CONTEXT - LFTVD.pptx
SOCIAL AND HISTORICAL CONTEXT - LFTVD.pptx
 
Paris 2024 Olympic Geographies - an activity
Paris 2024 Olympic Geographies - an activityParis 2024 Olympic Geographies - an activity
Paris 2024 Olympic Geographies - an activity
 
Organic Name Reactions for the students and aspirants of Chemistry12th.pptx
Organic Name Reactions  for the students and aspirants of Chemistry12th.pptxOrganic Name Reactions  for the students and aspirants of Chemistry12th.pptx
Organic Name Reactions for the students and aspirants of Chemistry12th.pptx
 
CARE OF CHILD IN INCUBATOR..........pptx
CARE OF CHILD IN INCUBATOR..........pptxCARE OF CHILD IN INCUBATOR..........pptx
CARE OF CHILD IN INCUBATOR..........pptx
 
Mastering the Unannounced Regulatory Inspection
Mastering the Unannounced Regulatory InspectionMastering the Unannounced Regulatory Inspection
Mastering the Unannounced Regulatory Inspection
 
Concept of Vouching. B.Com(Hons) /B.Compdf
Concept of Vouching. B.Com(Hons) /B.CompdfConcept of Vouching. B.Com(Hons) /B.Compdf
Concept of Vouching. B.Com(Hons) /B.Compdf
 
Presiding Officer Training module 2024 lok sabha elections
Presiding Officer Training module 2024 lok sabha electionsPresiding Officer Training module 2024 lok sabha elections
Presiding Officer Training module 2024 lok sabha elections
 
How to Make a Pirate ship Primary Education.pptx
How to Make a Pirate ship Primary Education.pptxHow to Make a Pirate ship Primary Education.pptx
How to Make a Pirate ship Primary Education.pptx
 
Staff of Color (SOC) Retention Efforts DDSD
Staff of Color (SOC) Retention Efforts DDSDStaff of Color (SOC) Retention Efforts DDSD
Staff of Color (SOC) Retention Efforts DDSD
 
The Most Excellent Way | 1 Corinthians 13
The Most Excellent Way | 1 Corinthians 13The Most Excellent Way | 1 Corinthians 13
The Most Excellent Way | 1 Corinthians 13
 
A Critique of the Proposed National Education Policy Reform
A Critique of the Proposed National Education Policy ReformA Critique of the Proposed National Education Policy Reform
A Critique of the Proposed National Education Policy Reform
 
Kisan Call Centre - To harness potential of ICT in Agriculture by answer farm...
Kisan Call Centre - To harness potential of ICT in Agriculture by answer farm...Kisan Call Centre - To harness potential of ICT in Agriculture by answer farm...
Kisan Call Centre - To harness potential of ICT in Agriculture by answer farm...
 
Science 7 - LAND and SEA BREEZE and its Characteristics
Science 7 - LAND and SEA BREEZE and its CharacteristicsScience 7 - LAND and SEA BREEZE and its Characteristics
Science 7 - LAND and SEA BREEZE and its Characteristics
 
mini mental status format.docx
mini    mental       status     format.docxmini    mental       status     format.docx
mini mental status format.docx
 
Software Engineering Methodologies (overview)
Software Engineering Methodologies (overview)Software Engineering Methodologies (overview)
Software Engineering Methodologies (overview)
 
Model Call Girl in Tilak Nagar Delhi reach out to us at 🔝9953056974🔝
Model Call Girl in Tilak Nagar Delhi reach out to us at 🔝9953056974🔝Model Call Girl in Tilak Nagar Delhi reach out to us at 🔝9953056974🔝
Model Call Girl in Tilak Nagar Delhi reach out to us at 🔝9953056974🔝
 
Alper Gobel In Media Res Media Component
Alper Gobel In Media Res Media ComponentAlper Gobel In Media Res Media Component
Alper Gobel In Media Res Media Component
 
Industrial Policy - 1948, 1956, 1973, 1977, 1980, 1991
Industrial Policy - 1948, 1956, 1973, 1977, 1980, 1991Industrial Policy - 1948, 1956, 1973, 1977, 1980, 1991
Industrial Policy - 1948, 1956, 1973, 1977, 1980, 1991
 
Separation of Lanthanides/ Lanthanides and Actinides
Separation of Lanthanides/ Lanthanides and ActinidesSeparation of Lanthanides/ Lanthanides and Actinides
Separation of Lanthanides/ Lanthanides and Actinides
 
18-04-UA_REPORT_MEDIALITERAСY_INDEX-DM_23-1-final-eng.pdf
18-04-UA_REPORT_MEDIALITERAСY_INDEX-DM_23-1-final-eng.pdf18-04-UA_REPORT_MEDIALITERAСY_INDEX-DM_23-1-final-eng.pdf
18-04-UA_REPORT_MEDIALITERAСY_INDEX-DM_23-1-final-eng.pdf
 

Insuratest P&C Exam Mastery System .docx

  • 1. Insuratest 2022 Welcome to the Insuratest P&C Exam Mastery System portion of your INSURATESTexam preparation process;glad you are here. You will need a few things before we begin this final section.Cinnamon gum, yes cinnamon chewing gum, earplugs or headphones,and comfortable attire. Seriously, go to the store and buy a pack of cinnamon chewing gum, a .50 cent pair of earplugs and find the headphones that came with your smartphone; if you have noise canceling, one’s even better. (we will get to this in a bit). If you are not yet dressed comfortablyand sitting in a relaxed setting, please make that happen, then come back. Please turn off,and tune out all outside and inside distractions; everything; sports,reality tv, Facebook,texting, even that inner voice inside your head that keeps on buzzing. You know, the one relentlessly stressing out about the exam you’re about to ace! Let that voice take a nap. Go to a quiet environment, or visit a comforting place like your favorite coffee shop,conference room,or library. Go to where you know you will focus the best. In reality, we don’t go to study; we instead move towards learning. The deeperyou go towards studying, the more you will absorb this section’s content and retain what’s necessaryto ensure you pass. While in your preferred environment, try to block out all background noises with headphones or earplugs. Yes, earplugs are the best.50 cents you will ever spend on studying, far better and less expensive than a $4 latte’. Using earplugs while learning lets you hear yourself think, and this is paramount to your success.Earplugs prevent background noise from interfering with your ability to move deepertowards studying and support your ability to remain in study longer. We want you to bring a pair of earplugs to your test site and wear them during your exam. During the three-hour test, we do not want you to be auditorily distracted by other test takers or proctors walking around, talking, coughing, sneezing, or hearing sirens from the outside. Earplugs will prevent outside noises from breaking your concentration.
  • 2. Insuratest 2022 If you preferto listen to music, try listening exclusively to classical. Better yet, music was written explicitly during and closelyfollowing the baroque music era, years 1600 to 1750.Music written from this period carries a distinct mathematically progressive time signature to its melodythat is scientifically proven to boostbrain power when listened to while studying. Listening to classicalmusic while studying will help you focus naturally and will aid you in moving further towards study. Sorry “today’s hits” you gotta go for now. If you have a commercial-free Pandora radio account, try the “Classical for studying” station. Or put some Bach, Vivaldi, Chopin, and Mozart on your iPod. Get your cinnamon gum ready. Why Cinnamon? Chewing cinnamon gum while studying has also been scientifically proven to boostbrain power, narrow one’s focus,and stimulate the brain’s natural ability to tie a fragrance to content memorization and enhance study material recollection. Just smelling cinnamon boosts cognitive function and memory. We want you to chew cinnamon gum while taking your State Exam quietly. You will be amazed at how well it will help this course’s material circle back to you on exam day. Comfortable - Check Earplugs or Headphones & Classical Music - Check Cinnamon Gum - Check Worries left to rest - Check
  • 3. Insuratest 2022 Ok, let’s begin! Hopefully, by now, you are relaxed, in your perfectstudy environment, chewing cinnamon gum, listening to Mozart, and ready to dive right in. But wait, we need to explain one more thing. The INSURATESTstudy theory. Open your mind to this. It goes against all you have been pre-programmed to acceptregarding educational principles and exam preparation methodologies.We need to change the way you study and approach this exam permanently. Practice does notmake perfect; no, it does not. Practice makes patterns.Practice the wrong pattern, and you will have to practice approximately 85% more to achieve the same result naturally obtained by simply practicing the correct pattern in the propersequence. It is, however, the repetitive practice of executing a perfectpatternin an ideal environmentthatmakes perfect.Belowis how you practice perfectlyfor your State exam. DO NOT TEST YOURSELF now.That’s right, DO NOT APPROACH THIS AS A TEST because it’s a repetitive factual recognition and acceptance exercise.Accepting this is paramount; the right answers are correct, they are Bright and Bold, and right in front of you. All you need to do is read them Backward and forwards, even out loud to yourself.Practicing this section perfectlyas a tool will ensure you recall what is correct on exam day.
  • 4. Insuratest 2022 Simply read the question(s) and the correctanswer(s). Then re-read the correctanswer(s) and the question(s). If you want to get fancy, try and make two questions out of each question in your mind by rewriting the question using the answer and the original question as part of the answer. Here is an example of the INSURATESTstudy theory in action. (YOUR SCREEN IS NOT BROKEN. THE COLORS ARE THIS WAY BY CHOICE, soon to be explained). “Assignment” as an insurance term refers to A. The agent assigned to an individual insured B. The insurer’s right to subrogate a claim settlement from a third party C. The transfer of an insured’s interest in a policy to another party D. The insured’s right to change issuing company's mid-term without paying a short rate cancellation fee The same question rewritten in your mind The transfer of an insured's interest in a policy to another party refers to A. The agent assigned to an individual insured B. The insurer’s right to subrogate a claim settlement from a third party C. Assignment D. The insured’s right to change issuing company's mid-term without paying a short rate cancellation fee A little secretabout“Practice Tests.” Students are set up for failure by being forced to take standardized insurance practice tests throughout educational development,at an insurance school, in the pre-license training courses,or in a workbook. The insurance exam simulator programs are no better. They all accomplishthe exact wrong thing.
  • 5. Insuratest 2022 By taking quizzes and tests with 10, 50, or even 150 questions on them, students spend 85% of their time regardless,reading and studying wrong answers, essentially inundated with erroneous statements and “noise.” It is confusing you beyond confuzzlement while essentially trying to learn a new language, the insurance language, in only fifty-two hours. It’s not rocket science,just basic math. If you spend 85% of your time looking at, reading to yourself,or out loud, essentially memorizing error- filled statements,you’re more likely to recall what's wrong on exam day. Do you have time for that? No wonder it is the case that 61% of all who take this exam do not pass on their first attempt. Could it be that they spent 85% of their preparation time focusing on wrong answers? They could have spent time with family, friends,working, or doing what makes them happy. Math doesn’t lie. Did you know the answeryou picked,whetherrightor wrong,you are 90% more likely to recallon exam day? What if every answer you rememberedwas the correctone? What if that was all you were re- programmedto see? Now we want you to accept what is right in the format presented here in the Insuratest P&C Exam Mastery System. You must focus 100% of your time absorbing and recognizing what is true and correct in the question format. So that you will, with absolute certainty, recall what is right and accurate when it matters mostwhen taking your state licensing exam. Messagefrom our founder: “I’ll let you in on a little secret.I did not pass this test the first time I took it. Even after attending a ten-day, 52-hour insurance schoolclass, having a great instructor, and studying for 80 hours plus, not passing was my result. I was part of the 60% statistic. Having to call my Dad, a successfulinsurance agent for thirty years at the time, who had a desk waiting for me in his office since I had just graduated college two weeks prior, to tell him I had failed my exam was one of the hardest calls, I have ever had to make. (Waaaaay harder than cold calling).
  • 6. Insuratest 2022 Even after spending most of my college graduation money to attend insurance schooland “studying” for hours. I was among 61% left wondering if this was even in the cards for me. It was a crushing disappointment.One I desire for you to avoid. When I finally figured out “how to study,” I immediately went to work on re- engineering my study material to be viewed in the Insuratest P&C Exam Mastery System format. I re-approached this exam with what is now the INSURATESTstudy philosophy and my result on test day was staggeringly different. On exam day, I was the first to finish my exam in a room of 30 test takers. It took me 92 minutes to finish a three-hour test. The same test I had run out of time on two weeks prior. Taking my time, making sure every question was answered correctly, I hit the submit button with absolute certainty that I passed. (I ONLY SAW THE RIGHT ANSWERSON EXAM DAY,THEY POPPED OUT FROM THE COMPUTERSCREENAS IF THEY WERE IN BOLD FONT). Walking out of the exam room on that breezy San Francisco afternoon with a smile from ear to ear, doing a little fist pump, and honestly crying a few tears of joy made for one of the bestafternoons of my life. Helping you achieve this same success is our missionat INSURATEST. Welcome again to the Insuratest P&C Exam Mastery System Sessions, and thank you for choosing and trusting INSURATEST.com,“the insurance test program designed to ensure you pass.” Now let’s begin. FOCUS ON WHAT IS RIGHT (Hint it’s Bold Black and Yellow ). What is wrong is shaded lightly, so you won’t focus on or even read it. The material only looks and feels like test questions,so you’ll respectthem as such. But please,do not even read what’s not yellow or italicized bold;it’s only part of the illusion.…. Now do yourselfa gigantic favor.Keep readingand prove the INSURATEST study theory to yourself.
  • 7. Insuratest 2022 Let’s start by quickly Mastering Single, Double,and Triple Negative Questions.You can read/study as many questions as you want, but if a double negative question’s phrasing habitually messes you up, you’ll pick wrong answers on exam day. The question’s direction must be determined first, even though you know the correctanswer. Scoring right hinges upon your ability to interpret the question's true intent. This might be the differencebetweenpassing and needing to try again. Words and phrasesto look out for and whatis their Mathematical/linguistic property,either negative (-) or positive (+). Commit these to memory. (-) Except, (-) Not, (-) Not Included, (-) Not True, (-) False), (-) None of the following, (-) None of the above, are typically excluded (-). (+) Included, (+) True, (+) Are part of the following, (+) Which of the following, (+) Out of the following, (+) Of the following—single Negative Question Examples. Consequential loss is considered an indirect loss (as compared to losses from the direct damage of a peril). Direct damages would be covered under different types of insurance, such as property/casualty or fire insurance. All of the following { are considered (+) } a consequential loss { except (-) }? A. loss of rental value B. damage to personal property C. extra expense incurred while rebuilding damaged property D. loss of business income while a building is closed for repairs With single negatives think of them as a U-turn. Since the (-) negative word occurred at the end of the question, the correct answer will be an untrue statement. (the exception, one that is not like the others, the one that doesn’t belong). Remember the kid's TV show Sesame Street’s “Which one is not like the others, which one just doesn’t belong” song? I know it’s childish, but it works. Sesame Street can help you solve single negative and double negative questions.
  • 8. Insuratest 2022 {“Which one is not like the others, which one just doesn’t belong.” Translating a single negative question is the same thing} It was looking like this question’s answer would be in the form of a positive phrase (+) until the last word except (-) came into play. Except caused the direction of the question to take a U-turn. The last (-, or +) word is always the U-turn with single negative questions. Please be sure to read the entire question twice if need be. The word except was the most important word to catch and was hidden in the end. With single negative questions (-), the last word wins. Extra expense insurance affords coverage for all of the following (+) if coverage is in force at the time of loss except (-)? A. Costs of setting up business in an alternative temporary location during the period of restoration B. Business income including payroll when a business is inoperable after incurring a loss (Why? This is covered by business income, not Extra Expense coverage). C. The extra expenses a business incurs to continue a business while damage to the premises is being restored D. None of the above
  • 9. Insuratest 2022 is Under a standard boat-owners policy. Which of the following liability coverage(s) is/are typically excluded (-): A. Vicarious liability B. Flotilla liability C. Medical liability D. General liability All of the following are part of (+) the standard business owners policy (BOP), except (-): A. Tenants improvements and betterments B. Business personal property coverage C. Professional liability coverage D. Business income coverage Double Negative Question Examples, None of the following statements (-) differentiates the perils of robbery and burglary in terms of coverage trigger except (-)? A. Requires taking property directly from a person B. Robbery is the broadest form of crime coverage C. Burglary requires physical signs of forcible entry or exit D. May involve use of force to cause bodily injury or instill a threat of violence (Hint a negative (-) + a (-) = a Positive (+). This is true in both basic mathematics and testing language. Since both negative words in this question (- None) and (- Except) cancel each other out, you are looking for a positive phrase like “included” or, in this case, a “true” phrase to answer this question. C. Burglary requires physical signs of forcible entry or exit is the distinction and true phrase. The coverage trigger between burglary and all other crime coverages is “forcible entry or exit.” Burglary, by definition, “must be evidenced by physical entry or exit into a
  • 10. Insuratest 2022 closed premise.” This is a rather easy definition question. Easy only if you navigated through the double negative mine field correctly. All of the following do not (-) describe a type of loss triggering a claim for monetary damages due to an insured un-willfully causing injury to another person or damage to another's property by a negligent act, except (-)? Remember a (-) + a (-) = a positive, so you are looking for the term that matches the definition stated in this question's text. A. Real property loss B. Physical loss C. Personal property loss D. Liability loss Commercial property theft, also called “fidelity coverage,” is provided by none (-) of the following coverage forms except (-)? A. Commercial property form B. Building and personal property coverage form C. Commercial Crime Insurance D. None of the above coverage forms afford coverage for theft All of the following are not (-) covered by a Commercial General Liability policy except (-)? A. Damage to “Your Work” B. Damage to “Your Product” C. Contractual Liability D. Premises operations All of the following are inland marine policy exclusions (-), except (-): A. Mechanical Breakdown B. Earth movement
  • 11. Insuratest 2022 C. Electrical breakdown D. Wear and tear Triple Negative Question Examples The liquor liability exclusion contained in a Commercial General Liability (CGL) policy excludes coverage (-) for none of the following (-) scenarios except (-)? A. Serving alcohol at a company picnic B. Buying a round of drinks for the sales staff at a company sponsored golf tournament C. The insured being involved in the alcoholic beverage business D. None of the above {Don’t worry about triple negative questions if they throw a few at you on exam day. Just keep using basic math to help. Excludes coverage (-) for none of the following (-) These cancel each other out, so now you are left with a single negative question except (-), and those are easy. CGL policies afford coverage for alcohol being served at company parties or corporate events: as long as the insured is not directly involved in selling alcohol or being in the alcohol service or distribution business. Therefore choice (C) represents the only exclusion and is the correct answer} All of the following are not (-) inland marine policy exclusions (-), except (-): A. Mechanical Breakdown B. Earth movement C. Electrical breakdown D. Wear and tear All of the following are not (-) inland marine policy exclusions (-), except (-): are not (-) + policy exclusions (-) = a +, which is then negated by the third negative ( except (-) making this in the end just another basic “except” (-) question.
  • 12. Insuratest 2022 Do not guess on single, double,or even triple negative questionsor succumbto their trickery. Use basic math to help you solve single or triple negatively phrased questions designed to confuse you. The key is to know which direction you are being asked to answer in the end, either - or +. Use your scratch paper to keep track of phrases to add, subtract, or cross out. Figure out your end direction, then simply choose the correct answer; it’s that easy. Good news Honestly, most double negatively phrased questions are easier content-wise than straightforward questions. The test writers only turn the phrasing into that of a single, double or triple negative in an effort to toughen up what would otherwise be an easy-to- average question. All of the following are covered (+) under the unendorsed ISO building and personal property coverage form except (-)? A. Light pole B. Gate C. Patio D. Signs All of the following are not (-) an additional coverage provided by an unendorsed HO-3 policy except (-)? A. Tenant legal liability B. Lawn, trees, shrubs and other plants C. TRIA D. Safe Burglary An agent’s appointment to an insurance company is terminated and the agent has no other appointments with any other insurance company at this time. Which of the following happens to the agent’s license automatically? A. The agent’s resident license remains in force until its term expires.
  • 13. Insuratest 2022 B. The agent’s resident license will be terminated and they will have to sign it over to the insurance commissioner C. The agent’s resident license becomes inactive. D. The agent's license will simply expire upon renewal. All of the following are (-)not required of a producing broker-agent before appointing an insurance solicitor (-)except? A. Must be an MGA B. Must hold a permanent license C. Must submit a notice of hiring to the DOI in 4 business days or less D. Must possess a certificate of authority It is proven that another person's negligent actions contributed to an injury What gives an insured the right to seek compensation? A. Coinsurance B. Tort law C. Waiver of subrogation D. Reinsurance Consequential loss is considered an indirect loss (as compared to losses from the direct damage). Direct damages would be covered under different types of insurance, such as property/casualty or fire insurance. All of the following are considered (+) a consequential loss except (-)? A. Loss of earnings B. Damage to personal property C. Extra expenses incurred while demolition of a partially burned structure
  • 14. Insuratest 2022 D. Extra expense incurred to move a business to a temporary location All of the following are true (+) of an Insurance Adjuster? A. Assists in settling claims on behalf of an insurance company B. Typically an employee of an insurance company therefore does not need a license C. Must be licensed D. Both A and B All of the following are true (+) of a Public Insurance Adjuster? A. Assists an insured in settling claims with an insurance company B. Typically an employee of an insurance company therefore does not need a license C. Must be licensed D. Both A and C Which of the following are (+) exclusive licenses? A. Bail B. Title C. Mortgage Guarantee D. All of the above
  • 15. Insuratest 2022 Special lines Surplus lines brokers: A. Transact business only with (represent )non-admitted insurers and must post a $50,000 Bond. B. Hold a valid surplus lines insurance license C. Transact business exclusively with foreign insurers D. Specialize in large commercial accounts only All of the following insurer settlement options under a personal auto policy are not available (-) except (-)? A. Induce the insured to purchase a replacement auto from a wholesaler B. Recommend the insured first attempt to settle with the negligent operator’s insurance as to avoid subrogation. C. Return stolen property to the first named insured at the address shown on the policy declarations page D. Demand the insured repair their vehicle at a repair shop on the insurer’s preferred provider list. All of the following are not (-) a common policy conditions for a commercial package policy except (-)? A. Waiver of subrogation B. Why are you reading this C. Pro-rata clause D. Inspections and surveys Which of the following is true (+) of an insurance agent? A. Represents the company and sells insurance
  • 16. Insuratest 2022 B. Must be appointed and may have binding authority. C. Cannot transact life or disability insurance. D. All of the above Shipowners may acquire coverage against vicarious liability claims due to the negligent operation of their vessel by their employee(s); which of the following coverage forms? A. Yacht owners policy B. Protection and indemnity insurance C. Ocean Marine coverage D. Longshoreman Indemnity act Which advisory organization(s) develop forms for the standard market? A. Acord Form processing service (AFPS) B. National Association of Insurers and Financial Advisors (NAIFA) C. Insurance Services Office (ISO) D. National Association of Insurance Commissioners (NAIC) Under the common policy conditions, who is authorized to make changes to a commercial policy? A. The producer B. The first named insured (with insurer's consent) C. Any insured named on the policy D. The insurance company None of the following are true (-) regarding the Law of Large Numbers except (-)? A. The larger the number of similar risks that are combined in a group, the more predictable future expected losses would be
  • 17. Insuratest 2022 B. Applicable use of the Law of Large Numbers is a characteristic of an ideally insurable risk C. Cannot be used to predict individual losses or profits D. All of the above The Insurance Commissioner-elect uses which of the following as a means to determine whether or not an insurer’s filed rate is excessive or unjustly discriminatory? A. Whether the proposed filed rate mathematically reflects the insurance company’s investment income B. The length of time passed since an insurer’s last full examination of its books and records by the office of the insurance commissioner C. If their rates are comparable to other insurers with the same business model D. In accordance with the guidelines set forth by the FDIC In comparing the personal property coverage of a dwelling policy to that of a standard ISO homeowners policy form, deduce which of the following is not (-) false (-)? A. Why are you reading this? B. Homeowners' policy claims are settled on an actual cash value basis, while dwelling policy claims are settled on a replacement cost basis C. There is no appreciable coverage difference D. Homeowners' off-premises coverage is 10% of the total property limit, while that of a dwelling policy is 100%. All of the following perils are not covered (-) on the basic form except (-) ? A. Terrorism B. Riot or civil commotion
  • 18. Insuratest 2022 C. Collapse D. Vandalism Which of the following statements best defines the standard commercial property policy “concurrent causation exclusion” and the effect it has on a potential claim settlement: A. Why are you reading this, B? If a loss occurs during the 90-day Binder issuance period, it is excluded C. If two or more perils happen in sequence to cause a loss, then only the damage caused by the first peril is covered D. If two or more perils happen at the same time to cause a loss, there is coverage for all of the damage if any of the perils are not specifically excluded Brian a landscape architect is brought into a suit claiming his design is solely responsible for the damage caused by improper draining at a claimant's property. What type of insurance would protect Brain For this type of mistake even if it is determined later that the suit is groundless? A. Professional liability B. Errors and Omissions insurance C. Why are you reading this D. Fidelity Bond How does an insured's unendorsed personal auto policy respond in the event an auto accident occurs in Florida a no-fault state that requires personal injury protection (PIP)? A. PIP coverage is provided, but the limit moves to Florida’s state minimum B. Coverage is excluded without the intestate travel endorsement C. Coverage automatically is broadened to provide PIP
  • 19. Insuratest 2022 D. The insured must purchase PIP from a licensed Florida agent within three business days of the accident if at fault ten days if not at fault. The named non-owner coverage endorsement form adds which of the following coverage(s)? A. Liability, medical payments, and uninsured motorist B. Other than collision, vicarious liability, and medical payment C. Bodily injury, medical indemnity, and other than collision D. Property damage only Ariana leaves her vehicle at an auto dealer's service center overnight to complete her scheduled maintenance. The following day a mechanic employed by the dealership makes a “Grande” mistake and is involved in an auto accident while test driving Ariana’s car. What coverage if any will Ariana’s personal auto provide for this claim? A. The personal auto policy is primary B. The personal auto policy is excess C. Coverage is excluded under the personal auto policy D. Why are you reading this All of the following are not true (-) except (-) when comparing the per accident coverage limit difference(s) between auto policies with single limits vs. split limits? A. The per accident single limit covers to all claims, while the split limit covers medical payments exclusively B. The per accident single limit applies only to property damage, while the split limit applies to vicarious liability exclusively
  • 20. Insuratest 2022 C. The per accident single limit applies to property damage, while the split limit applies to bodily injury exclusively D. There is no difference All of the following are not considered (-) special damages except (-)? A. Compensation for time away from work B. Compensation for a claimant's pain and suffering C. Compensation for a claimant's loss of hearing D. Compensation for a claimant's loss of limb All of the following are part of (+) the standard business-owners policy (BOP), except (-): A. Tenants improvements and betterments B. Business personal property coverage C. Professional liability coverage D. Business income coverage Use this chart to help illustrate the different insurer Cancellation, Renewal and Nonrenewal notices as well as the advance “days” notice they require for Personal Auto, Homeowners and Commercial insurance questions. Test your recollection with the following questions. Personal Auto Homeowners Commercial Cancellation for Non-payment 10 days 10 days 10 days Notice of Non- Renewal 30 days 30 days 60 days Notice of Renewal 20 days 30 days 30 days
  • 21. Insuratest 2022 An insurer decides to non-renew a commercial auto policy how many days written notice must they provide the insured? A. 10 B. 20 C. 30 D. 60 An insurer decides to renew a commercial auto policy how many days written notice must they provide the insured? A. 10 B. 20 C. 30 D. 60 An insurer decides to non-renew a personal auto policy how many days written notice must they provide the insured? A. 10 B. 20 C. 30 D. 60 An insurer decides to renew a personal auto policy how many days written notice must they provide the insured? A. 10
  • 22. Insuratest 2022 B. 20 C. 30 D. 60 An insurer decides to non-renew a Homeowners policy how many days written notice must they provide the insured? A. 10 B. 20 C. 30 D. 60 An insurer decides to renew a Homeowner policy how many days written notice must they provide the insured? A. 10 B. 20 C. 30 D. 60 An insurer must afford a personal auto policy owner _______ days written notice of cancellation for non-payment of premium? A. 10 B. 20 C. 30 D. 60 An insurer must afford a Homeowners policy owner _______ days written notice of cancellation for non-payment of premium? A. 10 B. 20 C. 30
  • 23. Insuratest 2022 D. 60 An insurer must afford a commercial policy owner _______ days written notice of cancellation for non-payment of premium? A. 10 B. 20 C. 30 D. 60 Of the various types of cancellations which one occurs when the insurer keeps the earned, and returns the unearned premium? A. Pro-rate/rata B. Flat Rate C. Rescind/Recission D. Short Rate Of the various types of cancellations which one occurs when the insurer keeps the earned,and returns the unearned minus a fee for admin charge or overhead expense? A. Pro-rate/rata B. Flat Rate C. Rescind/Recission D. Short Rate Of the various types of cancellations which one occurs when the insurer voids the contract and returns all premiums? A. Pro-rate/rata B. Flat Rate
  • 24. Insuratest 2022 C. Rescind/Recission D. Both B and C {Flat Rate and Rescind/Recission mean the same thing: when the insurer Voids the contract and returns all premiums as if the contract never existed} An agent who redirects premium funds held in a fiduciary account to his or her personal account is guilty of? A. Fraud B. Theft C. Falsifying D. Perjury {Regarding Fiduciary responsibility please note that the term Fiduciary refers to premiums held in trust. Any form of commingling or embezzling is a form of theft. Please also remember that premium refunds coming from the company must be returned within 25 days. While premium refunds coming from the agent of record must be returned within 15 days. None of the following are true (-) of an insurance broker except (-)? A. An Insurance Broker represents the client and secures coverage B. Must post a $10,000 bond C. Cannot bind coverage or transact life or disability insurance D. All of the above Standard commercial lines insurance policies contain Interline endorsements (such as cancellation notices or nonrenewal provisions). These apply or could apply to: A. More than one coverage part of a package policy B. Why are you reading this C. To liability coverage only
  • 25. Insuratest 2022 D. Only to waivers of subrogation issued to certificate holders by endorsement All admitted insurers are required to maintain an investigative unit to question which of the following? A. Possible non-actuarially based discrimination B. Unfair employment and hiring practices C. Possible fraudulent claims submitted by insureds D. All vicarious liability claims Under a standard boat-owners policy. Which of the following liability coverage(s) is/are typically excluded (-)? A. Vicarious liability B. Flotilla liability C. Medical liability D. General liability The vacancy permit endorsement covers which of the following perils? A. Vandalism B. Building glass breakage C. Sprinkler leakage D. All of the above None of the following (-) are considered special damages except (-)? A. Loss of hearing B. Mental anguish
  • 26. Insuratest 2022 C. Lost wages D. Reassignment Insureds Lori and Daniel rent a convertible while vacationing in hawaii what coverage, if any, will respond to a hypothetical physical damage claim under their personal auto policy? A. They need to purchase physical damage coverage at the car rental agency B. Why are you reading this C. No coverage available D. The broadest coverage on any auto shown in the declarations of Daniel & Lori’s personal auto policy All of the following are not (-) one of the principal rating factors under Proposition 103 permitted for personal auto policies except (-)? A. Merit rating B. Safety driving record C. Credit report D. Using prior policy as a method of rate factoring Which of the following is the defining characteristic of a Fraternal Insurer? A. Charitable B. Non-participating policies C. Owned by shareholders D. Shareholders receive taxable dividend (when declared) None of the following (-) are eligible for a Businessowners policy except (-)? A. An auto dealer B. A furniture store C. A casino
  • 27. Insuratest 2022 D. A banking institution Which of the following is not listed (-) on a Commercial General Liability declarations page? A. Pollution liability limit B. General Aggregate Limit C. Rented to you sub-limit D. Personal and advertising injury limit None of the following (-) are forms of Professional liability insurance except (-)? A. Medical Malpractice B. Errors and Omissions C. Directors & officers and Employer Practices Liability D. All of the above Subrogation as an insurance term is summarized by which of the following statements? A. The insurance policy liability limits respond separately to each claimant B. The insurer’s right to recover payment from a responsible third party C. The insured’s right to receive PIP coverage at no extra charge while traveling across state lines D. The transfer of insurable interest in a policy to another party when a vehicle is sold {Remember subrogation is basically substitution. It is the transference of an insured's claim rights of financial recovery, to a third party. Ex: an insured is rear ended. Their insurance company (insurer) settles the claim, even though the accident was proven not to be their insureds fault. After paying said claim, the non-fault driver’s insurer (insurance company) then asks their insured to sign a notice of subrogation that basically says: “I have been paid (indemnified) for this claim (my loss) and I sign over my rights of subrogation (right to collect again) to my insurance company.”
  • 28. Insuratest 2022 This is done so that the non-fault representing insurer, who has paid out on said claim, may then be paid back through subrogation, by the at-fault driver's insurer. By the insured (non-fault driver) signing over their claim rights after being indemnified by their insurer, the principle of indemnity is followed. For by signing over rights of subrogation after indemnification (being paid for a loss) prevents an insured from collecting twice on a claim. So in essence subrogation supports indemnity, the principle of insurance} An insured submits a claim for stolen jewelry and signs the claim form. Later the claim is proven to be fraudulent. The insured may be guilty of_____? A. Attempted theft B. Collusion C. Perjury D. An insurance related misdemeanor All of the following statements regarding business owners policies is/are true (+) except (-)? A. Insureds have the benefit of additional coverages that might otherwise be overlooked B. Combine coverage for special advantages C. Assembles the basic coverages required by a business owner in one bundle. D. All of the above _________________ are very specific as far as money in a claim settlement is concerned. Example: It is exactly known how much a car door costs to replace, including labor. A. Stated Values B. Specific Awards C. General Damages D. Special Damages
  • 29. Insuratest 2022 Which of the following deals with lost wages or salary? A. Punitive Damages B. Specific Award C. General Damages D. Special Damage Non-specific losses, with non-specific claim settlements such as pain and suffering or mental anguish are characteristic of? A. Punitive Damages B. Specific Award C. General Damages D. Special Damage Awards using a monetary punishment to set an example to the industry? A. Punitive Damages B. Specific Award C. General Damages D. Special Damage {Example: In 1992, 79-year-old Stella Liebeck bought a cup of coffee at a McDonald’s drive-thru in Albuquerque New Mexico and spilled it on her lap after opening it up to put cream and sugar in it. She sued McDonald’s and a jury awarded her nearly $3 million in punitive damages for the burns she suffered. Stella definitely got her groove back with that settlement! Ever notice now at the McDonalds drive through they ask if you want cream and sugar at time of order, so they may put it in for you. Risk management effectively eliminated the need for their customers to ever open their hot coffee cup lids, especially while driving.}
  • 30. Insuratest 2022 Which of the following items of personal property are covered by an unendorsed personal auto policy and if so up to what limit? A. Electronic items up to $500 B. Non commercial use electronic equipment up to $1500 C. All personal property left in vehicle owned by the first named insured up to $500 D. There is no coverage for personal property The following are characteristics of a Stock Insurer (+)? A. Non-participating policies B. Owned by shareholders C. Shareholders receive taxable dividend (when declared) D. All of the above Which of the following is the defining characteristic of a Reciprocal Insurer/Inter- insurance Exchange? A. Also known as participation policies B. Owned by policyholders C. Policyholders receive tax-free dividends (when declared) D. Unincorporated/Attorney-in-fact Lloyds of London is a marketplace with physical facilities where business is conducted by groups of Lloyd’s members known as ___________? A. Captives B. Wholesales C. Syndicates D. intermediaries
  • 31. Insuratest 2022 None of the following are true (-) of Mutual Insurers except (-)? A. Also known as participation policies B. Owned by policyholders C. Policyholders receive tax-free dividends (when declared) D. All of the above All of the following are inland marine policy exclusions (-), except (-): A. Mechanical Breakdown B. Earth movement C. Electrical breakdown D. Wear and tear A home’s foundation is damaged during an earthquake, minutes later a gas line ruptures causing an explosion. The home burns to the ground. Which coverage if any is afforded by the unendorsed HO-3 policy form in force at the time of loss? A. There is no coverage for damage to the foundation directly caused by the earthquake, only for the damage caused by the explosion and subsequent fire B. There is no coverage for any loss directly caused by an earthquake in the unendorsed HO-3 policy form C. The HO-3 is a named peril policy form and does not cover earthquake or damages caused by explosion. D. The policy covers all of the damage since explosion is an insured peril At what time during the sales process is a fire and casualty agent required to inform a prospective insured of any conditions which may be precedent to coverage regarding a homeowners policy? A. At quote B. At binding
  • 32. Insuratest 2022 C. At application D. At policy issuance The insurer’s right to recover its claim settlement payment to their insured from a negligent third party is the definition of the following term: A. Subrogation B. Inherent claim rights C. Arbitration D. Reassignment What is the minimum discount awarded to "good drivers" as a percentage for meeting the qualifications? A. 10% B. 15% C. 20% D. 25% Interline endorsements such as notices of cancellation may pertain or could pertain: A. To more than one coverage part of a package policy B. Exclusively to ocean marine coverage C. To general liability and advertising injury only D. Only when required by a certificate holder An armored vehicle transporting an insured's bank deposit is robbed at gunpoint. In order for the insured's loss to be covered what type of insurance would need to be in force at time of loss? A. Safe burglary B. Inland marine coverage form C. Outside the premises-theft of money and securities
  • 33. Insuratest 2022 D. Robbery 10 days prior to cancellation for non-payment of premium a written notice is mailed to the insured by the insurer. Which of the following requirements regarding written notices by mail from an insurer to an insured is not required (-)? A. Notice must be sent with return receipt required B. Notice must be mailed to the residence or principal place of business C. Postage must be prepaid D. Why are you reading this In anticipation of an upcoming vacancy period at a seasonal business location the vacancy permit endorsement is added to a building and personal property coverage form. The basic policy's vacancy conditions as an interline endorsement is/are: A. Waived during the policy period B. Waived only for designated perils during the permit period C. Waived only for general liability claims during the policy period D. Waived during the permit period but only if a security system is installed What is the primary coverage differential between umbrella and excess liability policies? A. The umbrella has must follow form and be issued by the same character as the general liability policy B. The umbrella covers physical damage losses only not general liability losses
  • 34. Insuratest 2022 C. An umbrella policy may include coverage for perils not covered by the underlying policy; subject to a self insured retention limit (SIR). While excess liability policies follow form and may offer monoline coverage. D. The umbrella affords defense costs only if the insured wins the suit the excess liability does covers defense costs regardless In which of the following situations is off-premises coverage void under the business personal property coverage form? A. In a salesperson’s luggage B. While in a delivery truck C. At a convention center D. In a salesperson’s vehicle Which of the following policy forms affords an insured coverage from any loss except that of which arises from specifically excluded perils? A. Specified cause of loss B. Stated Value C. All-risk, Special Form, or open peril D. Named Peril The basic boat owner's policy form is similar to what other policy in terms of coverage? A. The personal auto policy B. A yacht owners policy C. A dwelling policy D. An inland marine floater policy
  • 35. Insuratest 2022 Please get up, walk around,come back in 10-20 min to keep it fresh. Vito’s Pizzeria owner and first named insured Vito Corleone does not own the building his pizzeria operates out of. Any improvements and betterments he makes to the building he leases are covered under what section of his (BOP) business owner's policy? A. Inland Marine B. Business personal property C. Rented to you sublimit D. “Forget about it”
  • 36. Insuratest 2022 A loss ratio is calculated by which of the following formulas? A. Expenses divided by premiums B. Losses and expenses divided by premiums C. Losses divided by premiums D. None of the above An expense ratio is calculated by which of the following formulas? A. Expenses divided by premiums B. Losses and expenses divided by premiums C. Losses divided by premiums D. None of the above A combined ratio is calculated by which of the following formulas? A. Expenses divided by premiums B. Losses and expenses divided by premiums C. Losses divided by premiums D. None of the above Which of the following are reasons Insurer’s purchase reinsurance: A. Unearned premium reserve reduction B. To protect themselves from catastrophic losses, assisting in profit stabilization C. Increases the insurer's underwriting capacity D. All of the above Which if the following is true with regard to private passenger vehicle inspection requirements outlined in the insurance code?
  • 37. Insuratest 2022 A. Requires a vehicle inspection only if both comprehensive and collision coverages are carried B. The insurance code requires a vehicle inspection for any vehicle with a gross weight exceeding 4,000 pounds C. The insurance code makes no mention of a vehicle inspection requirement D. No vehicle inspection is required if coverage D is added at policy inception What coverage if any would respond to an accident involving a named insured driving a company car who carries an unendorsed personal auto policy? A. Coverage (B) Medical payments B. The unendorsed policy form excludes this usage C. Why are you reading this D. Coverage (A) General Liability Claims for legal liability may stand on the basis of all of the following (+) except (- )? A. Breach of contract B. Vicarious liability C. Tort law D. Negligence With regard to admitted insurers actuarial methodologies, all of the following are not true (-) except (-)? A. They may use race, color, religion, or national origin as a condition to apply higher rates as long as their means are actuarially sound B. They may not use race, color, religion, or national origin as a condition to apply higher rates C. They may use any actuarially sound method when rating policies as long as they can be statistically proven D. They may use any actuarially sound method when rating policies as long as they can be statistically proven and are filed with the department of insurance
  • 38. Insuratest 2022 Which of the following statements best defines policy “assignment”: A. A transfer of legal rights under, or interest in, an insurance policy to another party. B. A transfer of legal rights under, or interest in, an insurance policy to only the first named insured C. The insurer’s right to collect damages from a third party only if a waiver of subrogation endorsement is in force D. The insured’s right to change insurance companies mid term without having to pay a new policy fee What effect does the building ordinance or law exclusion/limitation have on both commercial and homeowner’s property coverage policies? A. It automatically voids replacement cost coverage for both commercial and residential structures built built before the year 1985 B. It excludes that part of the loss resulting from the enforcement of a building ordinance or law for commercial buildings and limits it’s allocated loss dollar by a percentage of total property coverage limit on homeowner’s policies. C. It excludes that part of the loss resulting from the enforcement of a building ordinance or law or and limits it’s allocated loss dollar by a 5% of total liability coverage limit. D. Does not include coverage to upgrade handicap access building code enforcement While property insurance covers debris removal for a portion of property damaged by a covered peril, it doesn’t cover demolition expenses for an
  • 39. Insuratest 2022 undamaged portion of a building that has to be removed. Which Ordinance or Law coverage part provides this coverage? A. Part A of Ordinance or Law provides this coverage. B. Part B of Ordinance or Law provides this coverage. C. Part C of Ordinance or Law provides this coverage. D. Part D of Ordinance or Law provides this coverage. Which of the following statements differentiates the perils of robbery and burglary in terms of coverage trigger? A. Requires taking property directly from a person B. Robbery is the broadest form of crime coverage C. Burglary requires physical signs of forcible entry or exit D. May involve use of force to cause bodily injury or instill a threat of violence Coverage for which of the following is provided by coverage(s) A and/or B of the ISO commercial general liability (CGL) policy form? A. Vicarious liability B. Workers compensation statutory limit C. Pollution Liability D. Advertising injury Which type of loss describes that of a business exposed to the death, injury or disability of employees? A. Personal loss exposure B. Personnel loss exposure C. Fraternal loss D. nolo contendere
  • 40. Insuratest 2022 An Ohio based companies only option available for the purchasing workers compensation insurance is through the Ohio Bureau of Workers Compensation. Which of the following statements is true of Ohio’s workers compensation: A. Ohio has a competitive state fund B. Ohio has a shared market fund C. Ohio has a monopolistic state fund D. Ohio has a guaranty fund The act of an insurer making a false statement either written or oral with the intent to deceive to any examiner lawfully appointed to examine the insurer’s books and records is defined by which of the following? A. A barring act B. Concealment C. An unfair practice D. Fraud Which of the following is true of extra expenses under the business income coverage form? A. Excluded B. Covered without limit during the period of restoration C. Payments are executed if the restoration period exceeds 6 months D. Covered up to 25% of the policy limit specified in the declarations As an insurance agent which endorsement would you recommend your insured add to their personal auto policy to obtain liability coverage for their all- terrain vehicle (ATV)? A. Coverage is only available through the purchase of a recreational vehicle stand alone policy B. ATV Flotilla liability
  • 41. Insuratest 2022 C. Miscellaneous type vehicle endorsement D. Recreational/seasonal vehicle endorsement What legal recourse in action, in terms of claim settlement allotment does an injured party to a contract have if the other party is found to be in breach of said contract? A. Claim rights only if a waiver of subrogation is in force prior to execution of said contract B. Damages for breach, reasonable attorney fees and costs C. Damages for breach, reasonable attorney fees and costs less the self insured retention limit D. Damages for breach exclusively Inherently possessing the possibility of financial loss resulting from ownership of property is known as? A. The principal of insurance B. Forecasting C. Insurable interest D. Adverse selection (All of the following (+) are representative of areas market regulation covers, except (-)? Following the rule you are seeking a negatively phrased outcome (The “exception” in this case the one which does not belong : A. Employee training B. Monitoring and preserving the financial solvency of insurance companies C. Regulating and standardizing insurance policies and products D. Controlling market conduct and preventing unfair trade practices
  • 42. Insuratest 2022 Protection and Indemnity insurance, covers all of the following (+) occurrences except (-)? A. Loss of life, injury and illness of crew, passengers and other persons B. Damage to docks, buoys and other fixed and floating objects C. Mutiny and misconduct by crew D. The cost of reasonable measures an insured must take to prevent or mitigate damage at time of loss Which of the following occurrences would be covered if an insured purchases a scheduled personal property endorsement (i.e.: personal articles floater) from his agent to append to their already in force previously un-endorsed homeowners policy form? A. Wear and tear, gradual deterioration, latent defect, insects or vermin B. Inherent vice C. Breakage of glassware caused by an earthquake D. Personal property on display at a county fair Coverage for transportation expenses in an unendorsed personal auto policy form is provided in the event __________________ losses occur and coverage is in force at time of loss? A. Theft losses only B. Other than collision losses only C. Collision and other-than-collision losses D. Transportation expenses are not available in the unendorsed policy form An implied warranty in an insurance contract qualifies as which of the following?
  • 43. Insuratest 2022 A. An representation B. An Oath C. Res Ispa Loquitur (It speaks for itself) D. A Jurat {Remember a representation is the best of one’s knowledge or belief/promise.} None of the following (-) are characteristics of a Representation except (-)? A. Representations may be either oral or written. B. A Representation may qualify as an implied warranty. C. A representation may be changed before the policy goes into effect and agrees with its assertions or stipulations D. All of the above Which of the following is/are true of a/an Estoppel? A. An Estoppel essentially removes a real or potential liability for the other party in the agreement. B. In a settlement between two parties, one party might, by means of an Estoppel, relinquish its right to pursue any further legal action once the settlement is finalized. C. The Estoppel can either be in written form or some form of action. An Estoppel essentially removes a real or potential liability for the other party in the agreement. D. A legal doctrine restraining a party from contradicting its own previous actions if those actions have been reasonably relied on by another party. {Example: An insurer that has habitually accepted late premium payments from an insured may be estopped from later canceling said policy on the grounds of nonpayment because the insured has been reasonably led to believe that late payments are acceptable.} Which one of the following is characteristic of the the loss prevention method “Risk Avoidance”? A. A homeowner decides to buy a trampoline B. A homeowner installs carbon monoxide detectors C. A new homeowner decides to fill in the backyard swimming pool
  • 44. Insuratest 2022 D. A homeowner purchases flood insurance What is the coverage limit specific for tapes, records, disks, and other media in the “Coverage for excess sound producing equipment, audio, visual and data electronic equipment, and tapes, records, discs and other media endorsement” is found in a personal auto policy (PAP)? A. $500 B. $200 C. Coverage is D. $1,000
  • 45. Insuratest 2022 Coverage in the inland marine policy form contains all of the following perils as exclusions (-) , except (-)? A. Damage from insects, vermin or rust B. Wear and tear C. Earth movement D. Inherent vice Which of the following is/are true of a waiver? A. A waiver essentially removes a real or potential liability for the other party in the agreement. B. In a settlement between two parties, one party might, by means of a waiver, relinquish its right to pursue any further legal action once the settlement is finalized. C. The waiver can either be in written form or some form of action. A waiver essentially removes a real or potential liability for the other party in the agreement. D. All of the above { At the most basic level a Waiver effectively gives up a right} All of the following (+) will influence an entity's ability to hold a license except (-)? A. Resignation by or termination of a key employee B. Cessation of a co-partnership C. Cessation of an LLC D. Cessation of an association Which of the following best defines a claim loss reserve as it relates to the insurer?
  • 46. Insuratest 2022 A. The total incurred amount less the total paid amount B. The total paid amount less the total paid amount C. An estimate of the value of a claim or group of claims not yet paid D. Losses in excess of policy limits less self insured retention fee Which of the following defines a case reserve as it relates to the insurer? A. An estimate of the amount for which a particular claim will ultimately be settled or adjusted or reconciled B. The total incurred amount less the total paid amount C. The total paid amount less the total paid amount D. Losses in excess of policy limits less self insured retention fee The miscellaneous type vehicle endorsement when added to a personal auto policy affords an insured which of the following? A. Liability and Medical Payments coverage exclusively B. Physical damage coverage exclusively C. Passenger Liability exclusively D. All coverage under the personal auto policy None (-) of the following define Market Conduct Regulation except (-)? A. Examiners from the Market Conduct Regulation sector physically audit insurance carriers only to review their business practices as they relate to licensing, claims, tax payments, rates and form approval, advertising and sales aids, underwriting and rating, policyholder service, and complaint handling B. Examiners from the Market Conduct Regulation sector physically audit insurance agencies, insurance companies, etc., to review their business practices as they relate to licensing, claims, tax payments, rates and form approval,
  • 47. Insuratest 2022 advertising and sales aids, underwriting and rating, policyholder service, and complaint handling. C. Examiners from the Market Conduct Regulation sector physically audit non- admitted insurance companies exclusively, to review their business practices as they relate to licensing, claims, tax payments, rates and form approval, advertising and sales aids, underwriting and rating, policyholder service, and complaint handling D. None of the above Which of the following best defines “liberalization clause” and is accurate in terms of it’s sectional placement in an insurance policy? A. In umbrella liability insurance, a clause specifying that coverage will be as broad as that provided by the primary liability policies, without additional premium, found in the policies declarations section. (I.E. policy follows form) B. In umbrella liability insurance, a clause specifying that coverage will be as broad as that provided by the primary liability policies, without additional premium, found in the policies insuring agreement section. (I.E. policy follows form) C. In umbrella liability insurance, a clause specifying that coverage will be as broad as that provided by the primary liability policies, without additional premium, found in the policies conditions section. (I.E. policy follows form) D. None of the above How far in advance must an insurer provide written notice to an insured before it may legally cancel for non-payment of premium? A. 4 days B. 10 days C. 25 days D. 30 days Which of the following is true of an agent who possesses a valid inactive California insurance license? A. The agent may review quotes with a prospect but does not have binding authority
  • 48. Insuratest 2022 B. The agent may not transact insurance as there is no notice of appointment on file with the department of insurance but may keep their license inactive if their licensing fees are paid and all continuing education requirements are met. C. Why are you reading this D. None of the above Jeffrey lives in San Francisco by himself, takes public transportation to his job and does not own a car. He does however frequently drive his girlfriend Joy’s car. Which of the following policies should Jeffrey purchase to protect himself legally and how would Joy’s policy respond in the event of an occurrence? A. Named non-owned coverage policy. Joy’s policy would be secondary Jeffrey’s would be primary B. Named non-owned coverage policy. Joy’s liability policy would be primary Jeffrey’s would be secondary C. Driver other car endorsement. Joy’s policy would be secondary Jeffrey’s would be primary D. Miscellaneous type vehicle endorsement. Joy’s policy would be secondary Jeffrey’s would be primary An endorsement to an HO policy form that provides coverage for boats one owns, rents or borrows is defined as which of the following? A. Watercraft endorsement B. Miscellaneous type vehicle endorsement C. Recreational vehicle endorsement D. Flotilla liability endorsement Which of the following HO policy forms offers open peril coverage (A Structure Coverage) for an insured's home? A. HO-3
  • 49. Insuratest 2022 B. HO-4 C. HO-6 D. HO-8 All of the following do not (-) describe a type of loss triggering a claim for monetary damages due to an insured un-willfully causing injury to another person or damage to another's property by a negligent act except (-)? Remember a (-) + a (-) = a positive so you are looking for the term that matches the definition stated in this question's text. A. Real property loss B. Physical loss C. Personal property loss D. Liability loss Which of the following are required of an insured after suffering a collision loss regardless of fault per a personal auto policies conditions section? A. Make no attempt to protect the insured property from further loss B. Allow the insurer to inspect the damaged property before it is repaired C. Notify the police if damages are in excess of $500 D. Go to the repair shop the claims adjuster recommends Similar to aircraft and boatowner’s policies which coverages are typically provided in a boat owner's policy? A. Flotilla liability B. Inland Marine, hull, and protection & indemnity C. Physical damage, liability, and medical payments to driver/operator and passengers D. Professional liability, flotilla liability and medical payments to driver/operator and passengers
  • 50. Insuratest 2022 Although different in design in terms of coverage what is similar between a California HO-2, HO-4, and HO-6 policy? A. Cause of loss coverage A form perils B. Coverage C policy limits C. Cause of loss coverage C form perils D. None of the above Commercial property theft also called “fidelity coverage” is provided by none (-) of the following coverage forms except (-)? A. Commercial property form B. Building and personal property coverage form C. Commercial Crime Insurance D. None of the above coverage forms afford coverage for theft All of the following are not (-) covered by a Commercial General Liability policy except (-)? A. Damage to “Your Work” B. Damage to “Your Product” C. Contractual Liability D. Premises operations
  • 51. Insuratest 2022 Extra expense insurance affords coverage for all of the following (+) if coverage is in force at time of loss except (-)? A. Costs of setting up business in an alternative temporary location, during the period of restoration B. Business income including payroll when a business is inoperable after incurring a loss (Why? This is covered by business income not Extra Expense coverage). C. The extra expenses a business incurs to continue a business while damage to the premises are being restored D. None of the above Which of the following perils are covered under the ISO business income coverage form, choose the best answer? A. Those recorded on the declarations page B. Rental income C. Business Income D. Ordinary Payroll California Dreamin landscaping service owner Tom has the choice to purchase his company's workers compensation from a variety of different insurers presented by his insurance broker at time of policy renewal. He lives in a state which has _________________? A. A statutory state insurance fund B. A competitive state insurance fund C. A monopolistic state insurance fund D. A varietal insurance fund Which of the following coverages if any come standard with a DP-3 policy that are not part of a DP-1 policy form?
  • 52. Insuratest 2022 A. Fair rental value B. Personal property C. appurtenant structures D. Collapse To terminate a license that is in the possession of an employer which of the following must occur? A. The employer need only furnish a notice of employment, automatically terminating the licensee's license. B. The licensee must provide written notice of intent to the insurance commissioner C. The licensee must give written notice to the employer who must send written notice to terminate licensee’s license no later than 10 business days following termination of employment. D. The license cannot be terminated prior to the expiration date, it becomes inactive unless the licensee is charged with an insurance related misdemeanor or is convicted of a felony. To terminate a licensee’s license that is in the possession of a licensee which of the following must occur? A. Nothing, the licensee need only wait until the license expires for non payment of licensing fees B. Relinquish all appointments and return books and records to the commissioners office C. Deliver the physical document along with notice of relinquishment to the Commissioner D. None of the above Which of the following, if any, will cover a loss that occurs during the policy period but is discovered 75 days after the expiration date of a claims-made commercial general liability policy form. A. Inland marine permit endorsement
  • 53. Insuratest 2022 B. Supplemental extended reporting period endorsement C. The loss will not be covered because it was not reported during the policy period D. Value reporting form endorsement How would an ISO commercial building and business personal property coverage form, respond in the event a vandalism loss occurs and adjusters determine the building has been vacant in excess of 60 straight days at time of loss? A. The loss is covered 90 consecutive days is the exclusion coverage trigger B. The adjusted loss payment will incur a coinsurance penalty less the deductible C. The adjusted loss payment will incur a coinsurance penalty less the deductible D. The loss is excluded 60 consecutive days is the exclusion coverage trigger How would an ISO commercial building and business personal property coverage form, respond in the event a property loss occurs and adjusters determine the building has been vacant in excess of 60 straight days at time of loss? A. The loss is covered 90 consecutive days is the exclusion coverage trigger B. The adjusted loss payment will incur a coinsurance penalty less the deductible C. The adjusted loss payment will incur a coinsurance penalty less the deductible D. The adjusted amount of property loss will be reduced by 15% What is the main purpose of tort law as it pertains to insurance? A. To properly subrogate insurance claims made B. To protect an insured from fraudulent suits C. To prove negligence and determine responsibility for damages D. To increase the capacity for legal liability claims
  • 54. Insuratest 2022 Which of the following statements regarding building glass coverage is accurate with regard to the 2000 ISO editions of the basic, broad, and special causes of loss forms? A. Both the basic and broad causes of loss forms provide full coverage for building glass breakage due to vandalism in the same manner and to the same extent as damage to other types of covered property B. Both the broad and special causes of loss forms provide full coverage for building glass breakage due to vandalism in the same manner and to the same extent as damage to other types of covered property C. There is no coverage for building glass breakage due to acts of vandalism unless the vandalism and malicious mischief endorsement is appended to the policy D. There is no coverage for building glass breakage due to acts of vandalism available in either the basic, broad and special forms. All of the following statements reflect an insurer's use of the inspections and surveys condition found in a commercial package policy except (-): A. Recommend changes to mitigate future losses B. Guarantee safety regulation conformity C. Rate selection D. Make a decision on insurability The cost to replace an insured's damaged property, less depreciation as a means of indemnification is the formula for? A. Stated value B. Replacement cost value C. Actual cash value D. Appraised value An insured submits a fraudulent claim for a workplace injury. Following an adjuster's investigation, with the aid of worksite camera evidence it is determined the injury never occurred “at work”. Paid to date on this claim are the following:
  • 55. Insuratest 2022 Loss of wages $35,000, Chiropractic visits $10,000, physical therapy $15,000. What is the total fine imposed on the insured for fraudulently submitting a false claim to the insurer with the intent to deceive? {Remember, payout is 2 X’s the $ amount of the fraudulent claim submitted or $150,000 whichever is greater} A. $60,000 B. $100,000 C. $120,000 D. $150,000 According to the insurance code no insurer’s rate filing may remain in effect if it is considered to be any of the following, except (-): A. Having impartiality B. Possessing Inequitability C. Prejudicial in nature D. Actuarially insufficient Which of the following best defines “time element ” loss coverage and what are the two most common coverage form types? A. How long it takes to repair or replace the damaged property: Business interruption and third party indemnity B. How long it takes to repair or replace the damaged property: Seasonal indemnity and value reporting. C. How long it takes to repair or replace the damaged property: Business interruption and Extra Expense D. Why are you reading this None of the following statements (-) exemplify the rationale/purpose behind a commercial package policy inspections and surveys by an insurer except (-)?
  • 56. Insuratest 2022 A. To correct any OSHA violations B. To guarantee no losses will occur C. To determine the rate to be charged D. To justify a premium increase upon renewal _____________ damages provide a claimant with the monetary amount necessary to properly indemnify them, restoring them to the same financial condition that existed prior to suffering a loss and nothing more while ____________ damages are designed to punish the negligent party causing said loss? A. Punitive, General B. Compensatory, Punitive C. Special, General D. General, Special Damages awarded a personal injury claimant fall into two basic categories ______________ (also known as economic damages) and ______________ (also known as non-economic damages). Both types of damages are known as ____________ damages or actual damages? A. Special, General, Compensatory B. Special, General, Punitive C. General, Special, Compensatory D. General, Special, Punitive A/an _________ performs certain functions normally handled exclusively by insurers. Responsibilities include binding coverage, underwriting, pricing as well as appointing agents. They may also settle claims? A. Underwriter
  • 57. Insuratest 2022 B. Managing director C. Agency principal D. MGA (Managing General Agent) ______________ Is defined as a person (individual or corporation) who negotiates and binds ceding reinsurance contracts on behalf of an insurer? A. A credit insurance broker B. MGA (managing general agent) C. Insurance agency principal D. An TPA (Third party administrator) Motor carriers of goods are responsible for the cargo in their care custody and control up to a point. The extent or degree of that responsibility is established by a written contract or through a _____________. While ______________ limits a common carrier's liability subject to a dollar limitation on the value of the cargo or the dollar amount of any potential loss? A. Cargo Policy: inland marine policies B. Bill of lading: Release Bill of lading C. Value reporting form: Indemnity agreement D. Why are you reading this A personal auto policy conditions “Your Duties After Loss” section states an insured must notify the police in the event of the following occurrences? A. Loss caused by theft: Hit and run accidents B. Loss caused by theft: Accidents in excess of $750 in property damage C. Loss caused by theft D. Only need to notify the insurance carrier or agent
  • 58. Insuratest 2022 What coverage is/are available under the “extended non-owned” coverage endorsement when added to a personal auto policy to protect an insured who is furnished with a company car they drive off the clock for personal use as well? A. Coverage’s (A) Liability, (D) physical damage B. Coverages (A) Liability medical payments C. Coverage’s (A) Liability, (B) medical payments, (C) uninsured motorist D. Liability only Which of the following peril exclusions contained in the special form homeowner’s policy contract excludes coverage for mudflow? A. Flood B. Earthquake C. Landslide D. Subsidence A representation made by an insured during the application process may be withdrawn only__________________? A. any time within the 90 day binder period B. prior to insurance contract issuance C. after the effective date D. before a claim occurs __________ policies are a blend of__________ and __________ coverage in one plan and afford property coverage to farm equipment? A. Transit / standard / substandard
  • 59. Insuratest 2022 B. Truckers / inland marine / transit C. Farm / residential / commercial D. Inland marine / residential / commercial Which of the following coverages if in force at time of loss would respond to an insured's car being damaged by a hail storm? A. Hail Coverage endorsement B. Collision coverage (D) C. Comprehensive aka other than collision coverage (D) D. Miscellaneous type vehicle endorsement An insured may not purchase a homeowner’s policy form or must have their current homeowner’s policy form rewritten to a dwelling form for none (-) of the following reasons except (-)? A. Their home is for sale and listed on the MLS B. Their in laws move in to assist with childcare C. Their home is being renovated D. The owner no longer resides in the dwelling as it is rented to others In addition to third party bodily injury and property damage coverage afforded by coverage (A) all of the following (+) are covered in addition except (-)? A. Loss of wages if required to attend a hearing at the third party’s insurers request B. The cost to repair the third parties vehicle up to the in force coverage limit C. Prejudgment interest supplementary payments D. Why are you reading this { (-) remember the last word wins} A standard fire policy affords coverage (A) Dwelling and coverage (B) Other/Appurtenant structures. While a business owners policy covers (A)____________ and ___________ which includes ____________?
  • 60. Insuratest 2022 A. Building / Business Personal Property / Tenants improvements and betterments B. Building and Personal Property of Others / Care Custody and Control C. Rental agreement / Business Personal Property / Extra Expense coverage D. None of the above _______________ is something that has a tendency to self-destruct and is not covered (i.e. fruit may spoil during shipment)? A. Inherent Vice B. Jettison C. Barratry D. None of the above ___________ is to dump cargo overboard in attempts to save a ship from going under while __________ is/are illegal acts committed by the captain or crew without the ship owner’s knowledge? A. Barratry / Jettison B. Jettison / Barraty C. Barratry / Embezzlement D. Jettison / Embezzlement Which of the following coverage is afforded (+) an insured who purchases a DP3 policy that would otherwise not be covered with the purchase of a DP1? A. Sprinkler leakage
  • 61. Insuratest 2022 B. Collapse C. Additional living expenses D. All of the above An insured elects to choose a $500 other than collision deductible and a $1,000 collision deductible to cover his vehicle for physical damage. He is involved in an accident in which his vehicle suffers an “upset” overturning on highway 17 during a rainstorm. The vehicle suffers $8,000 in total damage. How much will the insurer pay for the loss? A. $7,500 B. $7,000 C. $500 D. $1,000 An insured elects to choose a $500 other than collision deductible and a $1,000 collision deductible to cover his vehicle. He is involved in an accident in which his vehicle suffers an “upset” overturning on highway 17. The vehicle suffers $8,000 in total damage. How much will the insured pay for the loss? A. $7,500 B. $7,000 C. $500 D. $1,000
  • 62. Insuratest 2022 Please get up, walk around,come back in 10-20 min to keep it fresh. Open-peril(cause of loss) also referred to as special form policies protect an insured against losses arising from? A. Perils contained in the exclusions section B. Perils involving open vacant land C. All perils, except for those which are specifically excluded (Typically Earthquake and Flood) D. Perils specifically named in the policy
  • 63. Insuratest 2022 Which of the following loss settlement options best describes an option the insurer may use to indemnity their insureds for property losses based on what the property in question could have been sold for at the time of loss? A. Actual cash value B. Stated value C. Market value D. Replacement value The “examination of commercial books and records” condition found in a commercial policy states the insurance company possesses the inherent right to audit an insured's books and records as they relate to their policy during the policy period or any time within three years after the policy periods end. None of the following reasons (-) are valid in terms of purpose to exercise this condition except (-)? A. To ensure the insureds business practice’s are not unethical B. To validate the proper amount of premium is charged for the risk C. To ensure OSHA regulations are not being violated D. To ensure no material warrants were violated post contract binding ___________ coverage is designed to cover property in transit over land, certain types of movable property (whether or not ever moved) as well as all risk coverage for endorsed personal articles such as a wedding rings and precious medallions? A. Ocean marine B. Flotilla liability C. Inland marine D. Miscellaneous cargo in transit coverage An insurance policy may not be transferred to another without:
  • 64. Insuratest 2022 A. Written consent of the agent B. Written consent of the insurer C. Oral consent of the agent D. Oral consent of the insurer The “Mexico coverage endorsement” will afford an insured temporary automobile coverage while visiting Mexico; barring they are in Mexico for no longer than a maximum of ________ days or less? A. 7 B. 10 C. 14 days or less D. 25 days or less A loss sustained policy form will pay for losses which occurred_________ the policy period yet are ___________ up to one year after policy _________. A. during, discovered, renewal B. prior to, discovered, renewal C. during, discovered, termination D. prior to, discovered, termination Why would an insured Dry cleaning business owner/operator need an inland marine policy form to cover his clients garments in his possession while being loaded into owned delivery vans on premises for scheduled delivery?
  • 65. Insuratest 2022 A. Clients garments in his care custody and control are only covered in the insured's building location or within 250 ft. from the insured premises B. Clients garments in his care custody and control are only covered in the insured's building location or within 100 ft. from the insured premises C. An insured dry cleaner will need only pollution liability inland marine insurance is covered automatically by this policy form D. Nne of the above What two sub-coverage lines are listed on a Commercial General liability policies declarations page (D.I.C.E)? A. Premises and operations and Products and completed operations B. Rented to you sub limit and professional liability coverage C. Medical payments to others and rented to you sub limit D. Products and completed operations and Medical payments to others Which of the following best describes Premises and operations one of the sub- coverages on a Commercial General liability policy's declarations page (D.I.C.E)? A. Covers damage caused by “your work” exclusively B. Covers losses at the location of the business C. Cover losses that occur away from the business D. None of the above Which of the following best describes Products Completed operations one of the sub-coverages on a Commercial General liability policy's declarations page (D.I.C.E)? A. Cover losses that occur away from the business B. Covers losses at the location of the business C. Covers damage caused by “your work” exclusively
  • 66. Insuratest 2022 D. None of the above Which of the following is/are not covered (-) by a Commercial General liability policy? A. Aircrafts B. Alcoholic Business C. Host liquor liability D. Both A and B Which of the following is/are (+) covered by a Commercial General liability policy? A. Aircrafts B. Alcoholic Business C. Host liquor liability D. Professional liability Truckers/motor carrier’s coverage minimum standard liability limits are? A. $ 750,000 B. $ 1,000,000 C. $ 3,000,000 D. $ 5,000,000 Truckers/motor carrier’s coverage minimum certain hazardous materials liability limits are? A. $ 750,000 B. $ 1,000,000 C. $ 3,000,000 D. $ 5,000,000
  • 67. Insuratest 2022 Truckers/motor carrier’s coverage minimum explosive materials liability limits are? A. $ 750,000 B. $ 1,000,000 C. $ 3,000,000 D. $ 5,000,000 None of the following (-) coverages represent the principal difference between an HO-3 and an HO-5 policy except (-)? A. Coverage A and B causes of loss (perils) B. Coverage C policy limits C. Coverage C causes of loss (perils) D. Coverage D policy limits None of the following (-) coverages will protect physicians, surgeons, and dentists for their professional activities except (-)? A. Commercial general liability B. Medical Malpractice coverage C. Professional liability D. Errors and omissions coverage Which of the following terms names the process through which a mutual insurer becomes a stock insurer sometimes called stocking or privatization? A. Mutualization B. Reorganization C. Stock Split D. Demutualization
  • 68. Insuratest 2022 The “Mexico coverage endorsement” will afford an insured temporary automobile coverage while visiting Mexico; barring they are in Mexico for no longer than a maximum of 10 days or less and do not drive more than ________ miles past the Mexico/U.S. border? A. 25 B. 50 C. 75 D. 100 An agent submits an application to an insurer he is not yet appointed with on behalf of an insured. The insurer accepts the application and issues the policy. What must the insurer now provide the insurance commissioner and in what time frame? A. An agent by no means may submit an application to a carrier not yet appointed with B. Send to the Insurance Commissioner a written notice of agent’s appointment within 10 days of receipt of said application C. Send to the Insurance Commissioner a written notice of agent’s appointment within 14 days of receipt of said application D. Send to the Insurance Commissioner a written notice of agent’s appointment within 30 days of receipt of said application Gurevich and Batstone agency represents Infinity Insurance Corporation. They may “express” the relationship between the two entities in all of the following (+) ways except (-)? A. Gurevich and Batstone agency underwriting for Infinity Insurance Corporation B. Gurevich and Batstone agency proudly representing Infinity Insurance Corporation C. Gurevich and Batstone agency dedicated sales associates for Infinity Insurance Corporation D. Why are you reading this Common policy conditions (D.I.C.E) include none of the following (-), except (-)?
  • 69. Insuratest 2022 A. Cancellation provisions/notices and who may make policy changes B. Examination of the insured's books and records and Insurer inspections and surveys C. Premiums charged for risk transference (insurance) as well as transfer of rights/assignment D. All of the above Which of the following per-day transportation expense i.e. rental car coverage limits are afforded an insured who has an unendorsed personal auto policy in force at time of loss? A. $20 B. $25 C. $35 D. None Of an MGA (Managing General Agent) all of the following are true (+) except (-)? A. Has the power to appoint, supervise and terminate agent-broker appointment contracts B. Has the express capability to either accept or decline risks based on actuarial principles C. Possesses a valid fire and casualty agent-broker’s license D. May not assist with the claims handling process Part 1 of a Workers Compensation Policy protects against statutory claims and it’s benefits include (+)? A. Medical with no time limit and no dollar limit as well as rehabilitation benefits
  • 70. Insuratest 2022 B. Disability Income at 66 ⅔% of a worker’s salary C. Death benefits in California: lump sum of up to $5,000 and income to the surviving spouse D. All of the above Part 2 of a Worker’s Compensation Policy protects against Common Law Claims and it’s benefits include (+)? A. Covers people with claims outside the scope of workers compensation part 1, with an unlimited lawsuit potential B. Subsequent/Second Injury Fund C. Pays the increase in compensation from a previous injury and or a secondary or subsequent injury D. All of the above Of the following loss exposures which one encompasses losses that of which may occur on an insured's land or in“real property” built on said land? A. Liability loss exposure B. Personnel loss exposure C. Farm liability D. Real property liability endorsement {OK here comes three sneaky questions…} Xavier is involved in a parking lot accident with Yoenis. Xavier’s vehicle suffers $6,000 in damage but adjusters determine that Xavier is 50% responsible for the accident. Xavier has a $1,000 collision and a $500 other than collision deductible How much would Xavier receive under comparative negligence rules? A. $ 0
  • 71. Insuratest 2022 B. $ 2,000 C. $ 4,500 D. $ 5,000 Xavier is involved in a parking lot accident with Yoenis. Xavier’s vehicle suffers $6,000 in damage but adjusters determine that Xavier is 49% responsible for the accident. Xavier has a $1,000 collision and a $500 other than collision deductible How much would Xavier receive under contributory negligence rules? A. $ 0 B. $ 2,000 C. $ 4,500 D. $ 5,000 Xavier is involved in a parking lot accident with Yoenis. Xavier’s vehicle suffers $6,000 in damage but adjusters determine that Xavier is 51% responsible for the accident. Xavier has a $1,000 collision and a $500 other than collision deductible How much would Xavier receive under contributory negligence rules? A. $ 0 B. $ 2,000 C. $ 4,500 D. $ 5,000 A flatbed truck with a gross vehicle weight of 9,500 lbs used to haul hay on Schweig ranch may be eligible for a personal policy because it is used in the business of: A. Commercial transport B. Hauling cargo for a fee C. Farming or ranching
  • 72. Insuratest 2022 D. Hauling agricultural goods for a fee Which of the following is true (+) of a Surplus lines broker? A. An Surplus lines broker represents non-admitted companies B. Must post a $50,000 bond C. Must be appointed and may have binding authority. D. Both A and B All of the following statements regarding the term “concealment” with respects to an insured's application are true (+) except (-)? A. Concealment of material relevant to the formulation of a contract once discovered are grounds for automatic declination by an insurer B. Concealment does not occur if the insured was not asked about a particular subject. i.e. no inquiry was made as to the fact that their building is located next to an explosives factory C. Concealment may be both intentional unintentional D. None of the above {Remember concealment is the intentional withholding of material information by an insured during the formulation of a contract. Although concealment is relevant to property and casualty insurance, it is most often associated with Life insurance contracts. If misrepresentation is discovered at any time during the policy period it may give the insurer grounds to rescind/flat rate cancel a policy as though it never existed. Example: an insured conceals the fact that he is a part time skydiving instructor on weekends and dies in a skydiving accident six months into a life insurance policy contract. The insurance company would most likely rescind the life insurance policy
  • 73. Insuratest 2022 purchased and return the premiums paid to the insured's beneficiary and not pay the death benefit on the grounds of concealment.} An insured owns a commercial warehouse building insured by an unendorsed commercial package policy. The building was vacant for a period of 75 consecutive days before a loss occurred. The policy will cover a property loss if caused by all of the perils listed below except (-): A. Vandalism and plate glass breakage B. Fire C. Windstorm D. Lightning Which of the following types of in force policies will pay for either a bodily injury or property damage loss that occurs, or is discovered during the policy period? A. Pollution liability policy B. Auto liability policy C. Claims made commercial liability policy D. Occurrence general liability policy What is the cut-off time period regarding claims submitted by an insured during the policy period under the unendorsed claims-made commercial general liability policy coverage form? A. 30 days B. 60 days C. 90 days D. 6 months An insured borrows his neighbor's truck to pick up a Christmas Tree how do the insureds and neighbors policies respond in the event an auto accident takes place?
  • 74. Insuratest 2022 A. Only the neighbor's policy provides coverage; there is no coverage by the driver's policy B. The neighbor's policy is secondary and the driver's policy is primary. Primary coverage follows the driver C. The neighbor's policy is primary and the driver's policy is excess. Primary coverage follows the car D. Not enough information regarding accident and or proof of negligence The following loss descriptions are all indicative of "ideally insurable loss exposure," except (-): A. Losses that are accidental B. Losses that are definite and measurable C. Large number of exposure units D. Losses that are catastrophic Vacancy conditions under a basic commercial property policy are waived for specified perils during the vacancy permit period barring the ___________ is added to a building and personal property coverage form at time of loss. A. Special form perils endorsement B. Extended form perils endorsement C. Vacancy permit endorsement D. Vandalism acts indemnity clause The commercial building and personal property coverage affords coverage for “your business personal property.” “Your” as a coverage definition references none (-) of the following except (-)? A. Only the first named insured listed in the policy declarations page B. Only the named insured(s) shown in the declarations
  • 75. Insuratest 2022 C. Owners of property in the care custody and control of the first named insured D. none of the above The process whereby insurers evaluate risks, decide which customers to insure, and what coverage(s) to offer; all while determining the risk transference rate to charge, based on sound actuarial principles is called? A. The law of large numbers B. Underwriting C. Rate Factoring D. Adverse Selection The liability coverage specific to a yacht owners policy is called? A. Slip liability B. Personal injury protection C. Protection and indemnity D. Professional liability An insured is in the business of pyrotechnic entertainment and is required to furnish proof of liability coverage prior to being booked for the State Fair’s 4th of july fireworks display. Since this risk is out of the ordinary and inherently dangerous in nature. Where would the insured go in order to obtain this type of liability coverage? Choose the best answer: A. Pyrotechnician labor union B. From an excess and or surplus lines broker C. From a nonadmitted insurer D. From an alien insurer Coverage afforded by the commercial equipment dealer’s property coverage form would include all of the the following (+), except (-)? A. Customer’s equipment in insured's care custody and control while in for service B. New equipment for sale
  • 76. Insuratest 2022 C. Motorized vehicles CHP approved for use on state highways D. Lease returned equipment for resale Of the following loss scenarios which of the following is not covered (-) by coverage (D) collision and other than collision coverage of an insured's personal auto policy? A. Tree limb falls in a severe storm damaging an insured vehicle’s hood B. Christmas presents left in the backseat of a vehicle overnight in a parking garage are stolen C. Vehicle suffers front end damage during an earthquake D. Vehicle suffers water damage during a severe rain storm What is the formula for net income? A. Total net loss minus expenses B. Total net gain less taxes and fees C. Total revenue minus total expenses D. Total revenue plus total payroll A standard ISO HO-3 coverage form also known as the “special form” policy affords coverage to an insured's real property on a/an __________ basis while it’s personal property is covered on a/an ____________ basis. A. All risk, broad form B. All risk, open peril C. All risk, special form D. Special form, open peril
  • 77. Insuratest 2022 Cargo coverage extends to none of the following (-) except (-)? A. Products being shipped B. covers the freight and reimburses the shipping charges in the event of a covered loss C. Inherent Vice D. Both A and B All of the following types of businesses are eligible for their business personal property aka “stock” to be covered under an inland marine commercial property coverage form except (-)? A. Stringed instrument repair shop B. Grocery Store C. Dry cleaners D. Jewelry store After running an annual DMV check and finding two major violations on the Cargo coverage extends to none of the following (-) except (-)? A. Products being shipped B. covers the freight and reimburses the shipping charges in the event of a covered loss C. Inherent Vice D. Both A and B Cargo coverage extends to none of the following (-) except (-)? A. Products being shipped B. covers the freight and reimburses the shipping charges in the event of a covered loss C. Inherent Vice
  • 78. Insuratest 2022 D. Both A and B An insured purchases a new vehicle and contacts his agent the next day to add the vehicle to his existing auto policy contract. What temporary coverage is extended from his existing auto policy during the time it takes to bind coverage? A. No coverage available must purchases temporary liability coverage from the dealer before driving off the lot B. The lowest coverage limits for any scheduled on his declarations page in force at the time he took ownership of the new vehicle. C. The broadest coverage for any scheduled on his declarations page in force at the time he took ownership of the new vehicle. D. Temporary coverage for liability only is afforded by his in force policy’s insuring agreement Which of the following homeowners coverage forms is most appropriate for a custom home with high end furniture and a state of the art sound-entertainment system inside? A. HO-3 B. HO-4 C. HO-5 D. HO-8 In addition to the mandatory online or classroom pre-license training program what else must an applicant complete whether it is for a life agent license, personal lines agent license or a fire and casualty license prior to applying for an actual state license? A. A course on ethical marketing and sales techniques B. A four hour course on ethical presentation standards C. None of the above
  • 79. Insuratest 2022 D. A 12 hour course on theinsur ance code and ethics When determining the value of a commercial property to estimate the 80% minimum coinsurance limit which of the following cost measures is implemented by the insurer? A. Replacement cost value B. Fair market value using comps as a value estimator C. The actual dollar amount to replace said commercial property at today’s construction labor rates & material costs D. The cost to replace said property less depreciation and a coinsurance trigger surcharge {You may see both variations of this question on your exam, both answers are correct as a “Tort is a legal wrong other than a crime or a breach of contract”} On the basis of tort law one may be held legally liable for a claim if any of the following are proven except (-)? A. Negligence B. Legal wrong C. Absolute liability D. Breach of contract On the basis of tort law one may be held legally liable for a claim if any of the following are proven except (-)? A. Negligence B. Crime C. Legal wrong D. Absolute liability
  • 80. Insuratest 2022 Alvin is involved an accident with Bernice. Alvins vehicle incurs $20,000 in physical damage. Adjusters determine Alvin is 25% responsible for the accident. How much would Alvin receive under comparative negligence rules? A. $ 0 B. $ 10,000 C. $ 15,00 D. $ 20,000 __________ is defined as the process an insurer undertakes to calculate an insured's risk transference premium based on sound actuarial principles? A. Rate-making B. Premium calculating C. Underwriting D. Loss assessment The unendorsed personal auto policy territory limits include none of the following (-) except (-): {Remember! (-) + (-) = + “What’s included”} A. Cayman Islands B. Toronto, Canada C. Tijuana, Mexico D. Cuba Which of the following losses caused by the negligent actions of an insured would result in a third party claim award for physical injury and/or property damage? A. Physical property loss B. Personnel property loss C. Liability loss D. Arbitration loss
  • 81. Insuratest 2022 Which of the following is true about the (CGL) Commercial General Liability coverage limit referenced on the declarations page? A. The CGL limit includes the products completed operations aggregate limit and advertising injury limit B. The CGL limit is a separate limit from the products completed operations aggregate limit and advertising injury limit C. The CGL limit is another name for per occurrence limit. D. The CGL limit is another name for the aggregate limit According to insurance code active agents and brokers must have their books and records made available to the commissioner’s office for examination ______________? A. Once every two years on the month of the agent or broker's license renewal B. Once a year on the month of the agent or broker's license renewal C. Within ten days after receiving a certified letter from the commissioner's office requesting said examination D. At all times and as often as the commissioner in his or her discretion deems appropriate How often may the insurance commissioner's office conduct an examination of the books and records of an active agent or broker per the Insurance code? A. Not less frequently than once every five years B. Reviews must be done annually C. Only in the event of a consumer complaint as often as necessary D. Once every three years Which of the following terms best describes the claim settlement option whereby an insured will replace an insured's lost, damaged or stolen property with that of the same like kind, quality, make and or construction?